V1 Block 3
A 24-year-old woman presents with a 3-month history of excreting fluid from her left breast. When her symptoms first started, she had noted clear fluid staining her bra, which recurred infrequently. If she compressed the left breast tissue around her nipple area, she could easily express clear fluid. Occasionally, with difficulty, she was also able to express the same fluid from her right breast. Medical and surgical history is positive for depressive disorder; it has treated with tricyclic antidepressants for the past 4 months; she had an appendectomy 2 years ago. Her menstrual history is regular, with dysmenorrhea present. She drinks alcohol occasionally. Family history is positive for diabetes. Question What history has the most significance in regard to this patient's condition? 1 History of tricyclic antidepressant use 2 Past history of appendectomy 3 Family history of diabetes 4 Menstrual history 5 Personal history of alcohol intake
1 History of tricyclic antidepressant use Explanation This patient seems to have galactorrhea. Causes of abnormal lactation can be broken down into the following categories: • Any disruption of production of dopamine from the hypothalamus will remove the inhibition of dopamine on the pituitary gland; this will result in increased prolactin production. Thus, craniopharyngiomas, empty sella syndrome, and anti-dopamine drugs (like metoclopramide and antipsychotics) can all antagonize the normal production of dopamine from the hypothalamus. • Fluctuating hormone levels of prolactin with a relative deficiency of estrogen and progesterone; therefore, pregnant women may lactate as early as the second trimester and continue to produce milk for up to 2 years after the cessation of breastfeeding. These fluctuating levels may also cause lactation during puberty and menopause. Similarly, stimulation of the breast (suckling, chest wall disorders, thoracic nerve irritation) and nipple tissue (poor fitting bras or irritating clothes) can increase the prolactin enough to stimulate lactation. Increased levels of TRH will stimulate prolactin production. Similarly, hypersecretion of cortisol and growth hormone may have associated hyperprolactinemia. • Chronic renal failure will decrease the excretion of prolactin and lead to hyperprolactinemia. • Serotonin also seems to play a part in increasing prolactin and patients on selective serotonin reuptake inhibitors (SSRIs, like fluoxetine) and cyclic antidepressants (like amitriptyline and clomipramine) have reported galactorrhea. • Contraceptive hormones can cause galactorrhea; this side effect seems to occur more often after discontinuation of oral contraceptive pills than during their use (similar to the withdrawal effect of precipitous drop of estrogen and progesterone that occurs after delivery). • Illicit drugs (amphetamines, cannabis, opiates, etc.) can all cause galactorrhea. Similarly, a number of herbs used in supplements have been reported to cause galactorrhea (fennel, anise, etc.). • Pituitary tumors, such as a pituitary prolactinoma, can cause galactorrhea. About 20% of females with galactorrhea have evidence of a pituitary tumor on testing, but this rate increases to 34% if the woman has concomitant amenorrhea. Some cancers oversecrete prolactin (e.g., renal cell cancers). In summary, the causes of galactorrhea are: • Physiologic conditions (14%) • Neoplastic process (Benign or malignant 18%) • Hypothalamic-pituitary disorders (non-prolactinoma/under <10%) • Systemic disease (<10%) • Medications and herbs (20%) • Chest wall irritation (<10%) • Idiopathic (35%) As can be seen from this breakdown, the most common cause of galactorrhea is idiopathic; this is a diagnosis of exclusion. The history must still be completed to rule out serious causes of galactorrhea. Tests to exclude hormone etiologies include prolactin level, human chorionic gonadotropin level, thyroid-stimulating hormone level, and BUN/Creatinine. In this patient, there is history of tricyclic antidepressant use, which points to the diagnosis of pharmacological galactorrhea. A menstrual history of amenorrhea, not dysmenorrhea, would be significant; a surgical history of a chest operation, not an appendectomy, would also be significant. A family history of thyroid disorder or multiple endocrine neoplasia type I, not diabetes, as well as illicit drug use, not alcohol intake, would have more significance.
A healthy couple presents for an evaluation prior to travel to the Dominican Republic; they leave in 1 month and will stay there 4 weeks. The CDC lists the Dominican Republic as a malaria-endemic area, but it is not considered to have a resistant strain of Plasmodium falciparum. Question What is the best advice for this couple? 1 "You should both take chloroquine weekly starting 1 week before travel." 2 "If you get any mosquito bites, return to the US immediately for malaria treatment." 3 "Be aware that malaria typically causes a mild 3-4 day self-limiting diarrheal illness." 4 "If you take prophylactic medications for malaria, you can discontinue them upon arrival if you have no symptoms." 5 "You should both pack prescriptions of high-dose amoxicillin in case malarial symptoms begin."
1 "You should both take chloroquine weekly starting 1 week before travel." Explanation When there is no drug resistance to Plasmodium falciparum (one of the Plasmodium protozoans that causes malaria), weekly chloroquine is the prophylactic drug of choice. It is well-tolerated and can be dosed once weekly rather than daily (hydroxychloroquine is similar for malaria prophylaxis). Travelers should be advised of the risk of malaria, chemoprophylaxis, and personal protection measures. The chloroquine should be dosed weekly starting 1-2 weeks prior to travel, continuing throughout travel, and discontinued 4 weeks after return from a malaria-endemic region. Atovaquone/proguanil, mefloquine, doxycycline, and primaquine can be considered for regions known to have resistant strains of P. falciparum. P. falciparum can be transmitted via mosquito bites. The risk of malaria infection rises with the number of mosquito bites, but the bite alone does not guarantee malaria, even in endemic regions. The couple should be educated on mosquito bite prevention for their travel. Malaria can be effectively treated in most countries, and patients should also be educated on when and how to seek care when traveling abroad. Malaria typically causes an influenza-like illness, with fevers, chills, malaise, headaches, and myalgias. Severe cases can progress to seizures and death. Prophylaxis, prompt evaluation, and treatment of malaria are essential for travelers. Malaria chemoprophylaxis should continue for 7 days to 4 weeks after return from travel. Continuation of treatment for 4 weeks post-return is recommended when using chloroquine, mefloquine, and doxycycline for malarial prophylaxis, and 7 days when using atovaquone/proguanil. While self-treatment is not ideal, the CDC recommends the use of atovaquone/proguanil (Malarone) if travelers develop symptoms without having utilized malaria prophylaxis. Self-treatment should be temporary, and travelers should be advised to seek medical care. Amoxicillin is not a treatment for malaria.
A 76-year-old man presents with a 2-day history of a red, hot, and swollen ankle. He denies fever, chills, headache, and any other symptoms. He is unable to put his weight on the foot, and he denies any trauma. He is a diabetic with poor glucose control. Laboratory analysis shows negatively birefringent crystals in the synovial fluid; there is no evidence of bacterial growth. The patient receives treatment, and his condition improves rapidly, but he has a similar attack 6 months later. Question What treatment is used in the prophylaxis of this condition? 1 Allopurinol 2 Sulfinpyrazone 3 Prednisone 4 Indomethacin 5 Colchicine
1 Allopurinol Explanation This patient has gout, which is a form of inflammatory arthritis; it is the body's response to the deposition of uric acid crystals in the joints. Gout presents as acute monoarticular arthritis in 90% of patients. In early gout, usually only 1 or 2 joints are involved (usually, the smaller lower extremity joints). Podagra, or inflammation of the first metatarsophalangeal joint, is the initial joint manifestation involved in about half of all cases, but podagra may also be observed in patients with pseudogout, reactive arthritis, gonococcal arthritis, psoriatic arthritis, and sarcoidosis. The attacks usually begin abruptly, and they can reach maximum intensity in 6-12 hours. The joints are red, hot, and extremely tender. Untreated, the characteristics of gout change over time and the attacks become more polyarticular. Although more joints may become involved, inflammation in a given joint may become less intense. Attacks occur more frequently and last longer; eventually, patients may develop a chronic polyarticular arthritis, which can be symmetrical and resemble rheumatoid arthritis. Tophi, which are collections of uric acid crystals in the soft tissues, occur frequently in untreated patients. They can be found in multiple locations, including the fingers, toes, and olecranon bursae; they can also be found along the olecranon, where they may appear to be rheumatoid nodules. Tophi tend to develop after 10 years in untreated patients who develop chronic gouty arthritis. Acute flares of gout can occur in situations that lead to increased levels of serum uric acid (e.g., the use of alcohol, overindulgence of certain foods rich in protein, trauma, hemorrhage, or the use of medications that elevate levels of uric acid). Allopurinol is the best answer for this patient who is elderly and diabetic with a history of gout. This drug is used in the prevention of future attacks; it is not used to treat acute flare-ups. Patients with frequent gout attacks may begin prophylactic treatment with allopurinol to prevent further episodes. Allopurinol blocks xanthine oxidase, thereby reducing the production of uric acid; therefore, it should be used in patients who overproduce uric acid. It is the most effective agent to lower serum uric acid levels. The side effects include rash, gastrointestinal symptoms, headache, urticaria, interstitial nephritis, and (rarely) hypersensitivity syndrome. It was originally developed as a chemotherapeutic agent. Alcohol can interfere with its effectiveness. Sulfinpyrazone, a uricosuric agent, is an alternative agent to aid in the prevention of attacks; however, it is not used as frequently since it can cause bone marrow suppression. Prednisone, a glucocorticoid, is similar in efficacy to other agents and has no greater risk of adverse effects in most patients, although other options may be preferred in patients with concomitant infection, prior glucocorticoid intolerance, diabetes, and those who are in a postoperative period in whom glucocorticoids may increase risk of impaired wound healing. NSAIDs have an anti-inflammatory effect that works by inhibiting cyclooxygenase, which acts to produce leukotrienes from arachidonic acid. Indomethacin is the drug of choice unless the patient is elderly or has underlying health problems, but other NSAIDs (e.g., ibuprofen, naproxen, sulindac, and ketoprofen) may be used. Aspirin cannot be used because it can elevate uric acid levels. Some of the side effects of NSAID therapy include gastropathy, nephropathy, and liver dysfunction. It can also cause fluid overload in patients with congestive heart failure, so its use should be limited in these patients. Colchicine is an antimitotic drug that is often used in patients who cannot take NSAIDs or corticosteroids. Due to its side effects (e.g., nausea, vomiting, diarrhea and, following intravenous administration, bone marrow suppression, renal failure, and death), however, it is not used as a first-line treatment.
A 48-year-old woman with a past medical history of obesity presents with a 4-hour history of moderate severity epigastric and right upper quadrant pain. The pain is intermittent and occurs in "waves." She also notes nausea, vomiting, and radiation of pain to the right shoulder. Her physical exam reveals normal vital signs, but tenderness is noted in the right upper quadrant. There is no guarding or rebound. A bedside ultrasonography is obtained; it reveals the following imaging. Refer to the image. Question What is correct regarding this patient's health maintenance? 1 Long periods of prolonged fasting should be encouraged. 2 A low-fat, low-cholesterol diet is recommended. 3 An immediate surgical evaluation is required. 4 This patient is considered a high risk for complications. 5 Fatty or greasy meals should be consumed.
2 A low-fat, low-cholesterol diet is recommended. Explanation This patient is presenting with signs and symptoms consistent with acute cholecystitis. Cholecystitis is related to conditions associated with biliary stasis, including debilitation, major surgery, severe trauma, sepsis, long-term total parenteral nutrition (TPN), and prolonged fasting. A low-fat, low-cholesterol diet can prevent gallbladder-related symptoms. Outpatient treatment may be appropriate for cases of uncomplicated cholecystitis. If a patient can be treated as an outpatient, discharge with antibiotics (such as levofloxacin and metronidazole), appropriate analgesics, antiemetics and definitive follow-up care. Criteria for outpatient treatment include afebrile with stable vital signs, no evidence of obstruction based on laboratory values, no evidence of common bile duct obstruction on ultrasonography, no underlying medical problems, advanced age, pregnancy, or immunocompromised condition, adequate analgesia, reliable patient with transportation, easy access to a medical facility, and prompt follow-up care. Gallstones are most common among overweight, middle-aged women, but the elderly and men are more likely to experience more serious complications from gallstones. Patients with gallstones should only consume clear liquids to promote gallbladder rest; fatty or greasy meals should be avoided.
A 28-year-old woman presents with an itchy throat, prolonged sneezing episodes, red and watery eyes, and inflamed nasal membranes. Her temperature is normal and a throat culture is negative. She most likely has allergic rhinitis. Question Which of the following factors is a common side effect of the drug diphenhydramine? 1 Weight gain 2 Sedation 3 Constipation 4 Jaundice 5 Addiction
2 Sedation Explanation Antihistamines are the drugs that have major applications in treating the symptoms of allergic rhinitis and urticaria. They may also be used in treating motion sickness and nausea. Some antihistamines are used in the treatment of insomnia because of their strong sedative properties; therefore, they also find their way into many over-the-counter sleep aids. The most common adverse effect observed with H1 receptor blockers is sedation. Other effects seen are tremors, blurred vision, lassitude, dizziness, fatigue, drying of the nasal passages, and dry mouth. Antihistamines can interact with other drugs, potentially leading to serious consequences, such as the potentiation of the effects of CNS depressants (alcohol, etc.). Monoamine oxidase (MAO) inhibitors can potentiate the anticholinergic effects of antihistamines. In spite of this, H1 receptor blockers are relatively safe. Chronic toxicity is rare; however, acute poisoning is common, especially among children, and it leads to dangerous effects, such as hallucinations, ataxia, convulsions, and if untreated, coma and cardiorespiratory collapse. Histamine is synthesized from the amino acid histidine by a decarboxylation reaction. It is found in practically all tissues; there are large amounts in the skin, lungs, and gastrointestinal tract. It is stored in high concentrations in the granules of mast cells and basophils. It is metabolized by monoamine oxidase enzymes and by methylation. Histamine is released from mast cells in response to immunoglobulin E (IgE)-mediated (immediate) allergic reactions and plays an important role in urtication, angioneurotic edema, seasonal (allergic) rhinitis (hay fever), and anaphylaxis. It may also play a role in the control of gastric acid secretions. Histamine exerts its effect by binding to 2 major types of cell surface receptors (H1 and H2). Recently, a third type (H3) has been discovered. H1 receptors are important in producing smooth muscle effects (e.g. in bronchial smooth muscle constriction leading to decreased lung capacity and symptoms of asthma, in intestinal smooth muscle contraction resulting in cramps and diarrhea, and in exocrine glands resulting in increased nasal and bronchial mucus secretion). Binding of histamine to the H1 receptor is also involved in vasodilation and increased permeability of capillaries leading to leakage of protein and fluid into tissues. Histamine binding to the H1 receptor causes bronchoconstriction by stimulating the activity of the polyphosphoinositol cycle (IP3 release) and vasodilation, probably by releasing endothelium-derived releasing factor (EDRF). The H2 receptor is involved in stimulating gastric acid secretion from parietal cells in the gastric mucosal glands. It also has a cardiac stimulatory effect. H2 receptor activity is expressed by activation of adenylate cyclase and concomitant increased cAMP levels. The H3 receptor seems to be active mainly in the CNS, where it may be involved in modulating histamine neurotransmission at some presynaptic membranes. Histamine itself has no therapeutic applications, but compounds that block its actions at the H1receptor (H1 receptor antagonists, antihistamines) are very important clinically. There are many kinds of H1 blockers from several different chemical classes. Refer to the image. H1 blockers do not have any effect on the formation or release of histamine from storage sites. They compete with histamine for receptor sites on target cells. H1 receptor blockers do not serve as histamine agonists due to the differences between their structures. Some H1 blockers may also show some affinity for the serotonin, adrenergic, and cholinergic muscarinic receptors. H1 blockers competitively inhibit the effects of histamine. All show varying degrees of sedative and anti-motion sickness effects in the CNS. Since some may act at other receptors (noted above), they may block muscarinic, α-adrenergic, and serotonin receptor-mediated effects.
A mother brings in her 5-year-old son due to papular and pustular lesions on his face. A serous honey-colored fluid exudes from the lesions. You suspect impetigo. A Gram stain reveals spherical gram-positive arrangements in irregular grape-like clusters. Question What organism is most likely causing this patient's condition? 1 Staphylococcus epidermidis 2 Staphylococcus aureus 3 Peptostreptococcus 4 Streptococcus pneumoniae 5 Haemophilus influenzae
2 Staphylococcus aureus Explanation The history and lab findings suggest the diagnosis of impetigo, in which Staphylococcus aureus is likely the causative organism. The most common causes of impetigo are usually more than likely Staphylococcus aureus, but also may be beta-hemolytic streptococcus group A. S. aureus causes inflammatory and toxin-mediated diseases. When a Gram stain has been completed, S. aureus will appear as spherical gram-positive cocci arranged in irregular grape-like clusters, as is described in this patient. Inflammatory diseases that can be caused by an active S. aureus infection include the following: • skin infections, including impetigo, furuncles, carbuncles, and cellulitis, surgical wound infections, eyelid infections, and postpartum breast infections • septicemia (sepsis), which can originate from any localized lesion, especially wound infection, or as a result of intravenous drug abuse • endocarditis on normal or prosthetic heart valves • osteomyelitis and arthritis, either hematogenous or traumatic • pneumonia in postoperative patients or following viral respiratory infection, especially influenza • abscesses (metastatic) in any organ Streptococcus pneumoniae are gram-positive lancet-shaped cocci arranged in pairs (diplococci) or short chains. On blood agar they produce alpha-hemolysis. Virulence factors of pneumococci are polysaccharide capsules. Pneumococci cause pneumonia, bacteremia, meningitis, and infections of the upper respiratory tract such as otitis and sinusitis. Mortality rate is high in elderly, immunocompromised (especially splenectomized), or debilitated patients. They should be immunized with the polyvalent polysaccharide vaccine. Peptostreptococci are anaerobic gram-positive cocci. It grows under anaerobic or microaerophilic conditions and produces variable hemolysis. Peptostreptococcus is a member of the normal flora of the gut and female genital tract, participating in mixed anaerobic infections of the abdomen, pelvis, and brain. Staphylococcus epidermidis is part of normal human flora on the skin and mucous membranes, but it can cause infections of intravenous catheters and prosthetic implants. This organism is particularly infectious in the inpatient hospital setting in patients with compromised immune systems. Gram stain of this bacterium reveals gram-positive cocci arranged in grape-like clusters. Although the Gram stain characteristics are similar to the correct answer, the patient scenario is inappropriate. Haemophilus influenzae is an incorrect choice in this scenario since observed Gram stain characteristics include gram-negative and rod-shaped.
A 52-year-old man presents with a 2-year history of worsening dyspnea and a 1-month history of dry cough. The patient gives no history of fever, chills, chest pain, or wheezing. History is significant for smoking (25 cigarettes/day for more than 22 years). Pulmonary function tests (PFT) reveal a decrease in forced expiratory volume in 1 second (FEV1) along with reduction of FEV1/FVC (forced vital capacity) ratio. The patient's chest X-ray is below. Refer to the image. Question These findings are characteristic of what condition? 1 Heart failure 2 Chronic bronchitis 3 Emphysema 4 Bronchiectasis 5 Pleural effusion
3 Emphysema Explanation The findings described suggest that the patient has emphysema. Emphysema is a chronic obstructive airway disease characterized by abnormal, permanent enlargement of the airways distal to the terminal bronchioles and destruction of their walls. Cigarette smoking is a major contributor to the development of emphysema. Other causes include air pollution, alpha-1 antitrypsin deficiency, and occupational exposure. The classical symptomatology includes prolonged progressive dyspnea with late-onset non-productive cough, occasional mucopurulent relapses, and eventual cachexia and respiratory failure. Patients are usually thin and have a barrel-shaped chest. There is tachypnea with pursed-lip breathing and use of accessory muscles; they may also adopt a tripod sitting position. A chest X-ray shows hyperinflated lungs with bullae, tubular heart, and flattened diaphragm. Forced expiratory spirometry quantifies airway obstruction. Pulmonary function tests (PFT) reveal a decrease in forced expiratory volume in 1 second (FEV1) along with reduction of FEV1/FVC (forced vital capacity) ratio. Treatment includes general measures, such as cessation of smoking, breathing exercises, symptomatic treatment like antibiotics to control secondary infection, bronchodilators, corticosteroids, and oxygen therapy. Lung volume reduction surgery and lung transplantation may be required depending on the severity. Heart failure presents with dyspnea, cough, orthopnea, weakness, abdominal discomfort, and edema in lower portions of the body. The chest X-ray may show cardiac enlargement. There may be pulmonary congestion with audible rales. Echocardiography confirms the diagnosis. Chronic bronchitis produces obstructive changes and bronchial inflammation. Chronic bronchitis is characterized by chronic productive cough for ≥3 months in each of 2 successive years for which other causes, such as infection with Mycobacterium tuberculosis, carcinoma of the lung, or chronic heart failure, have been excluded. Chest X-ray shows increased bronchovascular markings and cardiomegaly. Bronchiectasis is a condition characterized by abnormal permanent distortion of the conducting bronchi or airways, most often secondary to an infectious process. Patients with bronchiectasis often present with chronic cough with mucopurulent sputum production that lasts months to years. CT, especially HRCT (high-resolution computed tomography), helps in confirming the diagnosis. Pulmonary effusion is a collection of fluid in the pleural space. This may occur due to pleural disease and may be an incidental finding in a stable patient, but some patients may demonstrate respiratory distress or may be hemodynamically unstable. Small effusions are typically asymptomatic. Patients with larger effusions may present include shortness of breath, chest pain with deep inspiration, fever, and cough.
A 16-year-old girl presents with shortness of breath, wheezing in her chest, and swelling of the lips that started several minutes after she had a snack. She had similar symptoms several months ago at a Chinese buffet. She appears restless. Her face is erythematous, and her lips and tongue are swollen. Her blood pressure is 90/60 mmHg, and heart rate is 110/min, rhythm regular, with respirations 26/min and shallow. On auscultation, diffuse wheezing is heard. Question What is your next step? 1 Take arterial blood gases 2 Intubate the patient 3 Give Epinephrine 4 Give Corticosteroid 5 Give Epinephrine and beta blocker
3 Give Epinephrine Explanation Your patient most probably has acute anaphylactic reaction secondary to some component in foods; symptoms occurred shortly after the exposure, and she has hypotension, tachycardia, dyspnea, and swelling due to vasodilatation. Epinephrine should be administrated immediately at 1:1000 subcutaneously; it should be administered every 15 minutes until improvement. Arterial blood gases analysis for oxygen and carbon monoxide content are time-consuming analyses. In cases of acute dyspnea, such as this one, they should be replaced by pulse oxymetry. Endotracheal intubation would be indicated if the patient were to develop laryngeal edema with severe obstruction that is not improved after epinephrine. Corticosteroids would not resolve the problem immediately. In this case, they should be given intravenously together with antihistamines after epinephrine, and they should then be given orally for 1 week. It is not appropriate to give beta-blockers to slow the rhythm in anaphylactic shock. Beta-blockers will interfere with epinephrine, which acts on both alpha 1 and beta 1 receptors and displaces noradrenaline from presynaptic storages. Therefore, ephedrine and epinephrine will oppose one another (often in an unpredictable manner). Beta-blockers are actually contraindicated in people at risk for anaphylaxis.
A lactating woman with a 5-month-old baby presents with increasing unilateral breast discomfort over the past few days. A home monitoring of her temperature revealed a reading of 100.8°F (38.2°C). She has been unable to feed her child from the affected breast for the past 24 hours. Examination reveals a breast with a fluctuant mass. Refer to the image. Question What is the next best step in the management of this patient? 1 Oral flucloxacillin alone 2 Needle aspiration 3 Warm compresses with expression of milk 4 Oral flucloxacillin with IM gentamycin 5 MRI of the breast
2 Needle aspiration Explanation A lactational breast abscess is a relatively uncommon problem in feeding mothers, with an incidence of 0.1% (up to 3% in women with antibiotic-treated mastitis). Organisms ascend from the surface of the nipple and colonize the breast lobule. Milk serves as an excellent culture medium for the bacteria. Stagnation of milk due to cessation of feeding aids this process. What begins as a cellulitis/mastitis can thus evolve to form an abscess. The treatment of an abscess always involves drainage, regardless of its location; release of purulent material is key to resolution. With breast abscesses, needle aspiration under ultrasound guidance is the preferred initial intervention (and would then be followed by a course of oral dicloxacillin); however, if the overlying skin is compromised, or if needle aspiration and oral antibiotics are not sufficient for resolution, surgical incision and drainage may be recommended. Flucloxacillin is the antibiotic of choice for lactational abscess because it covers the Staphylococcus species, which are the usual culprits. However, antibiotics are only minimally effective without drainage of pus. At an early stage of development (when it is still considered mastitis), oral antibiotics alone may suffice; however, this would not be sufficient for this patient. Warm compresses with expression of milk is an adjunct to medical treatment, and it prevents the stagnation of milk in the breast. Gentamycin is used for gram-negative sepsis, but it is not used in abscesses. While ultrasound guidance is generally recommended during the needle aspiration procedure (to help delineate the abscess and ensure complete drainage), an MRI would be unnecessary.
A 40-year-old man presents with severe pain in his left eye, decreased vision, nausea, and abdominal pain. He denies recent trauma. He wears glasses for myopia. On examination, the patient's left pupil is moderately dilated and nonreactive. The cornea is "steamy" in appearance and the eye is red in general. Question What is the most likely diagnosis? 1 Conjunctivitis 2 Uveitis 3 Acute angle-closure glaucoma 4 Presbyopia 5 Corneal infection
3 Acute angle-closure glaucoma Explanation This patient has acute angle-closure glaucoma. The typical characteristics of this condition are all exhibited by this patient: steamy cornea, severe pain, blurred vision, dilated and nonreactive pupil. Conjunctivitis does not affect pupil size; rather, it is characterized by injection of the conjunctiva and either watery or purulent discharge. It may be viral, bacterial or allergic. A physical exam finding in a patient with uveitis would be a small pupil. In addition to these symptoms, patients may also present with abdominal pain. Presbyopia is loss of vision due to old age; it does not affect the pupil. It is a gradual decrease in vision rather than sudden. A corneal infection would cause circumcorneal injection and watery, or purulent, discharge.
A 28-year-old man presents for evaluation of a rash. His trunk is covered with a pink-red papulosquamous eruption with scattered discrete coppery papules on the palms of his hands and a few lesions on the soles of his feet. He is afebrile and has not experienced any illness in the past month that he can recall. Skin exam reveals soft, flat, moist papules scattered on his perineum. Question What laboratory test will yield the most information about this patient's condition? 1 KOH prep 2 HIV antibody 3 VDRL 4 Peripheral blood smear 5 Tzanck smear
3 VDRL Explanation Based on the clinical scenario, the most likely diagnosis is syphilis. The VDRL is a screening test for syphilis, which would be expected to be positive in this case. The patient's body rash specifically affecting the palms and soles strongly suggests secondary syphilis. Also, the perineal lesions suggest condylomata lata of secondary syphilis. The KOH prep would be helpful if the rash were due to a fungus. Although the patient should be encouraged to have an HIV antibody test, the scenario does not suggest HIV infection. The peripheral blood smear is not helpful here, and the Tzanck smear is used for the diagnosis of herpes simplex.
A 20-year-old woman presents with intermittent nose bleeds for the past 2 weeks. She also reports that her menstrual periods have increased in number in the past 2 months. She recently underwent surgery for small bowel resection and eats only one meal a day. Laboratory investigations reveal prolonged prothrombin time, prolonged activated partial thromboplastin time, and a normal platelet count. Question What is the most likely diagnosis? 1 Thiamine deficiency 2 Riboflavin deficiency 3 Vitamin K deficiency 4 Vitamin D deficiency 5 Iron deficiency
3 Vitamin K deficiency Explanation In cases of vitamin K deficiency, patients present with bleeding tendencies, such as epistaxis, menorrhagia, and hematuria. The prothrombin time (PT) and the activated partial thromboplastin time (aPTT) are usually prolonged. The platelet count, fibrinogen level, thrombin time, and bleeding time are normal. Causes include inadequate dietary intake combined with surgery, such as small bowel resection and the use of broad-spectrum antibiotics. Dietary sources rich in vitamin K include green leafy vegetables and liver. Vitamin B1 (or thiamine) deficiency results in beriberi, which is characterized by: a bilateral symmetric peripheral neuropathy beginning in the legs Wernicke-Korsakoff syndrome, which is comprised of nystagmus, ophthalmoplegia, ataxia, memory loss, and confabulation congestive heart failure with tachycardia, peripheral edema, and cardiomegaly In cases of riboflavin (or Vitamin B2) deficiency, patients present with angular stomatitis and cheilosis. On examination, they are pale, have atrophic glossitis, and the tongue may appear magenta. In cases of vitamin D deficiency, children can present with inability to walk unsupported due to muscle weakness and lower limb skeletal deformities, such as genu varum and genu valgum. In cases of iron deficiency, patients can present with feeling weak, dizzy, and tired; they may experience syncope. On examination, they have pale conjunctivae and koilonychia.
A 70-year-old woman presents with shortness of breath at rest over the past 3 days. She has found it difficult to walk short distances due to shortness of breath. Additionally, she is experiencing confusion, orthopnea, nocturnal dyspnea, and lightheadedness. She denies cough, fever, chills, diaphoresis, anxiety, chest pain, pleurisy, nausea, abdominal pain, vomiting, diarrhea, rashes, and syncope. On physical examination, the patient is short of breath, requiring numerous pauses during conversation. She is afebrile, but she is tachycardic, diaphoretic, and her extremities are cool. There is a diminished first heart sound, S3 gallop, laterally displaced PMI, bibasilar rales, and dullness to percussion and expiratory wheezing noted. An elevated JVD and 2+ pitting edema of the lower extremities is evident. Question What statement regarding this patient's condition is correct? 1 The most common etiology of this condition is infiltrative diseases. 2 Increased caloric and sodium intake improves patient outcomes. 3 Confusion is the most common presenting complaint in older patients. 4 Activation of the renin-angiotensin-aldosterone system occurs. 5 The electrocardiogram is the most useful diagnostic test.
4 Activation of the renin-angiotensin-aldosterone system occurs. Explanation This patient's presentation is most consistent with congestive heart failure (CHF). In developed countries, coronary heart disease with resulting myocardial infarction and loss of functioning myocardium (ischemic cardiomyopathy) is the most common cause of systolic heart failure. The most frequent cause of diastolic cardiac dysfunction is LVH, commonly resulting from hypertension, but conditions such as hypertrophic or restrictive cardiomyopathy, diabetes, and pericardial disease can produce the same clinical picture. The most common causes of CHF in the United States are CAD, systemic hypertension, nonischemic dilated cardiomyopathy, and valvular heart disease. Rare causes of dilated cardiomyopathy include infiltrative diseases (hemochromatosis, sarcoidosis, amyloidosis, etc.), other infectious agents, metabolic disorders, cardiotoxins, and drug toxicity. In CHF, lower cardiac output leads to a reduction of renal blood flow and glomerular filtration rate, causing sodium and fluid retention. In CHF, the renin-angiotensin-aldosterone system is activated, leading to further increases in peripheral vascular resistance and left ventricular afterload as well as sodium and fluid retention. Heart failure is also associated with increased circulating levels of arginine vasopressin, which also serves as a vasoconstrictor and inhibitor of water excretion. Dietary caloric restriction is particularly necessary in overweight patients because weight reduction lowers demands on the heart and can provide significant relief of symptoms. Restricting sodium helps reduce water retention, with a concomitant reduction in cardiac work. A moderate sodium restriction (1.5-2 g/day) is usually necessary to achieve therapeutically meaningful results.
A 53-year-old man presents with increased difficulty swallowing and occasional regurgitation of his meals. His symptoms have been occurring with greater frequency and severity over the last 4 months. He also has some shortness of breath but attributes that to his weight and lack of physical activity. His past medical history is remarkable for chronic heartburn, which he treats intermittently with over the counter antacids. He takes no regular medications has no allergies. He has not had any surgeries. He is not a smoker and he denies use of alcohol and drugs. He works as a building inspector, and he lives with his wife and children. The patient is obese, but the rest of his physical exam is normal. Blood tests, electrocardiogram, and chest X-ray are done in the clinic; they are normal. He is referred for endoscopy, and esophageal biopsy shows specialized intestinal metaplastic cells (of columnar epithelium). Question What recommendation would best prevent complications from this patient's current condition? 1 Avoidance of antacids 2 Referral for fundoplication 3 Regular exercise 4 Use of proton pump inhibitors 5 Weight loss
4 Use of proton pump inhibitors Explanation This patient is presenting with Barrett's esophagus, a type of chronic esophagitis in which the normal squamous epithelium is replaced with columnar epithelium. Barrett's esophagus is a complication of chronic gastroesophageal reflux disease (GERD) and can develop into esophageal adenocarcinoma. Use of proton pump inhibitors (PPIs) reduces the risk of cancer. In order to monitor for the development of cancer, a routine endoscopy should be periodically performed in patients with known Barrett's esophagus. While the antacids are not necessarily adequate to control the GERD and damage to this patient's esophagus, avoidance of antacids will have no role in preventing subsequent Barrett's esophagus complications, such as adenocarcinoma. While the acid reflux appears to be a major mechanism for damage to the esophageal tissue, anti-reflux surgeries do not appear to prevent complications from Barrett's esophagus and subsequent development of adenocarcinoma. This patient does not need referral for fundoplication (surgical "wrapping" of the stomach around the esophagus). Regular exercise would be helpful for this patient in general terms for cardiovascular health and weight reduction, but exercise does not play a significant role in reducing risks for Barrett's esophagus complications. Obesity is a risk factor for development of Barrett's esophagus. Other risk factors include male sex, smoking, and especially chronic GERD. Once Barrett's esophagus has developed, however, weight loss does not dramatically alter the risk of complications, such as adenocarcinoma. Weight loss should be recommended for any obese patient, but the PPIs are the best approach to reduce complications from his Barrett's esophagus.
A 47-year-old Caucasian woman, previously healthy, presents for evaluation of a 4-week history of dyspnea. Her symptoms started approximately 1 month ago when on vacation in Colorado. She initially attributed these symptoms to the altitude, but upon return to her hometown, she continued to have shortness of breath with mild activity, such as walking more than 100 feet, walking upstairs 1 flight, vacuuming, or sweeping. Her symptoms resolve with rest. She also reports mild exertional chest tightness and easy fatigability. She denies paroxysmal nocturnal dyspnea, orthopnea, edema, palpitations, and syncope. Her past medical history includes the usual childhood illnesses, no previous surgeries, and no known allergies. She takes a daily multivitamin and occasional Tylenol for headache. She is a non-smoker, rarely uses alcoholic beverages, and denies use of illegal or illicit drugs. Physical exam shows temp 96.9°F, pulse 80 and regular, respirations 16, and BP 136/82. O2 sat is 96% on room air. The patient is an alert Caucasian woman in no acute distress, with no obvious jugular venous distention; non-labored respirations; lung fields clear to auscultation and percussion; and no rhonchi, rales, or wheezes. Heart shows RV heave present; normal S1 with fixed, split S2 with prominent P2 component; and grade II/VI systolic murmur at the left upper sternal border at the second intercostal space. The remainder of a complete physical examination is within normal limits. CBC and BMP are unremarkable. Free T4 and TSH are within normal limits. EKG shows normal sinus rhythm with right ventricular hypertrophy, right atrial enlargement, and right axis deviation. There is an RSR in leads v1 and v2. Question After completion of diagnostic studies, this patient was found to have a congenital heart defect. What congenital heart defect was found? 1 Atrial septal defect 2 Coarctation of the aorta 3 Patent foramen ovale 4 Tetralogy of Fallot 5 Ventricular septal defect
1 Atrial septal defect Explanation The correct answer is atrial septal defect. Atrial septal defect (ASD) is an opening in the cardiac septum that divides the left and right atria. Left heart pressures are ordinarily much higher than those of the right heart. Thus, an ASD usually causes a left-to-right cardiac shunt. This results in chronic volume overload of the right heart chambers and delayed closure of the pulmonary valve, which in turn results in a widely split, fixed S2, a classic physical finding associated with ASD. The chronic volume overload causes increased right ventricular stroke volume across the pulmonic outflow tract, resulting in a systolic ejection murmur of the left second intercostal space, as was detected in this patient. Coarctation of the aorta is incorrect. Coarctation of the aorta is a narrowing of a segment of the aorta, typically distal to the origin of the left subclavian artery. If the coarctation is severe, symptoms (pallor, irritability, diaphoresis, or dyspnea) may be present in infancy. Aortic coarctation increases the left ventricular afterload, causing left ventricular hypertrophy. With coarctation, lower extremity (leg) blood pressures are lower than upper extremity (arm) blood pressures. In an adult, hypertension is frequently present and symptoms may include headaches. Patent foramen ovale is incorrect. A foramen ovale is a small opening in the atrial septum present in the fetal circulation prior to birth; it normally closes at birth. A patent foramen ovale (PFO) is one that did not close at birth. Up to 25% of people have a PFO. An isolated PFO is most often asymptomatic, and most individuals with PFO are unaware of their condition. A PFO may be associated with an increased risk of stroke, heart attack, or migraine headache. Tetralogy of Fallot is incorrect. Tetralogy of Fallot causes severe cyanotic symptoms in infancy and is usually diagnosed early in life. It consists of 4 heart defects: a large ventricular septal defect, pulmonary stenosis, right ventricular hypertrophy, and an overriding aorta. Treatment of tetralogy of Fallot is with open-heart surgery. Ventricular septal defect is incorrect. Ventricular septal defect (VSD) is an opening in the septal wall between the left and right ventricles of the heart. Large VSDs often cause symptoms in infancy or childhood and require surgical closure. Smaller VSDs may be asymptomatic or eventually close spontaneously. In adulthood, the cardiac murmur associated with VSD is typically prominent, holosystolic, and located at the left lower sternal border.
A 73-year-old man with no significant past medical history presents with a 1-month history of light-headedness, dizziness, and near syncope; it has been occurring in response to sitting up and standing from a supine position. He denies chest pain, palpitations, shortness of breath, cough, loss of consciousness, vision or speech changes, nausea or vomiting, numbness, tingling, paresthesias, and focal weakness. His physical exam is noteworthy for a drop in systolic blood pressure of 24 mm Hg from a supine to standing position. Question What clinical intervention should be recommended to this patient? 1 Begin clonidine or a diuretic 2 Fludrocortisone and compression stockings 3 Large, carbohydrate meal consumption 4 Sodium and water restrictions 5 Rise rapidly from seated and supine positions
2 Fludrocortisone and compression stockings Explanation This patient is presenting with signs and symptoms consistent with orthostatic hypotension, defined as a reduction in systolic blood pressure of at least 20 mm Hg or diastolic blood pressure of at least 10 mm Hg within 3 minutes of standing or head-up tilt on a tilt table. It is a manifestation of sympathetic vasoconstrictor (autonomic) failure. The use of compression stockings and medications such as fludrocortisone are of benefit in reducing orthostatic hypotension. Further interventions that should be made include the discontinuance of any provocative medications, if possible. These medications include clonidine, methyldopa, anticholinergics, opioids, muscle relaxants, tricyclic antidepressants, alpha adrenergic or beta-blocking medications, vasodilators, diuretics or quinidine, barbiturates, Parkinsonism agents, and phosphodiesterase type 5 inhibitors. Recommended dietary interventions include increasing salt and water consumption. This is indicated for 24-hour urinary sodium <170 mmol sodium in 24 hours. 1-2 grams extra should be supplemented per day; this should be avoided in CHF or edematous states. Water intake should be in excess of 64 ounces daily, with avoidance of alcohol and large carbohydrate-rich meals; instead, smaller and more frequent meals are recommended. Activities that are helpful in management include avoiding heat exposure or strenuous exercise, sleeping with the head of the bed slightly elevated, and rising slowly from the bed, allowing for equilibration. Other activity modifications such as avoidance of standing for long periods, isometric exercises, frequent movement or crossed legs while standing, avoidance of work with the arms above shoulder height and performance of dorsiflexion of the feet prior to standing are all considered supportive.
A 15-year-old boy presents with a diagnosis within the autism spectrum disorder. He is considered high-functioning. He attends public school, goes to daily cognitive/behavioral sessions, and attends weekly counseling sessions. His mother has noted a significant increase in episodes of what she can only describe as "temper tantrums" with symptoms of severe irritability and quickly changing moods. These occurrences have become so severe that they have begun to disrupt the patient's daily activities. Question What medication is used to treat irritability in children and adolescents with autism spectrum disorder? 1 Sertraline 2 Paroxetine 3 Atomoxetine 4 Clonidine 5 Risperidone
5 Risperidone Explanation The correct response is risperidone. Adolescents classified as having a diagnosis within the autism spectrum disorder (ASD) will have varying degrees of impairment in their social and behavioral function. Family education, behavioral and educational interventions, and counseling have a significant place in this treatment plan; pharmacotherapy may be considered, but it should be used as adjunctive therapy. In 2006, the United States Food and Drug Administration (FDA) approved risperidone, an atypical antipsychotic (in the oral disintegrating tablet form), for the symptomatic treatment of irritability in both children and adolescents with autism spectrum disorder. This is considered the first FDA-approved drug treatment for behaviors specifically associated with ASD, and it has been used off-label for many years for these symptoms. Other symptoms that risperidone could be considered to help treat in patients with ASD include aggression and deliberate self-injury. When beginning any pharmacotherapy for these patients, having a "start low and go slow" regimen is strongly recommended, with consistent follow-up visits for evaluation of alleviation of the symptoms. Selective serotonin reuptake inhibitor (SSRI) pharmacologic agents (e.g., sertraline or fluoxetine) are generally introduced to help alleviate anxiety symptoms. Although paroxetine is a type of SSRI that is used to treat patients with depression, obsessive-compulsive disorder, anxiety disorder, post-traumatic stress disorder, or premenstrual dysphoric disorder. Patients who express symptoms consistent with hyperactivity and inattention should be treated with methylphenidate, atomoxetine, or clonidine.
A 3-year-old boy has been found face down in a swimming pool during the summer. He was unresponsive when removed from the water and CPR was started by a neighbor. The child remains apneic and pulseless when the EMS team arrives. Resuscitative efforts, including tracheal intubation and intraosseous epinephrine, are continued during transport, but ECG shows asystole and he remains pulseless. Despite continued resuscitative efforts, he remains asystolic. Question What would be the most appropriate action concerning his mother in this situation? 1 Ask a staff member to direct the mother to a waiting room until you can talk to her alone. 2 Describe the resuscitative process in detail to ensure she understands all that has been done. 3 Tell her there have been stories of children surviving submersion accidents. 4 Begin to introduce the possibility of a poor prognosis. 5 Try to obtain a detailed accounting of how the child got into the pool.
4 Begin to introduce the possibility of a poor prognosis. Explanation This child is unlikely to respond to resuscitative efforts because he continues to be in pulseless arrest despite basic and advanced life support. Pediatric studies in the pediatric intensive care unit (PICU) show that children requiring specialized treatment for drowning experience at least a 30% mortality rate; an additional 10-30% experience severe brain damage. Successful resuscitation after prolonged submersion has been documented in only very small children pulled from very cold water. The mother should be gradually prepared for the eventuality of a poor outcome. If family members are not interfering with resuscitative efforts and are not causing harm to themselves, it is acceptable for them to stay if they choose. This will also help the mother grasp the reality of a child's unexpected death and will allow the opportunity to say goodbye. While it is helpful and appropriate to let the mother know the interventions that have been or are being undertaken, it is not appropriate to stop to provide a detailed description, especially not using complex terminology. Since this child is unlikely to respond to resuscitative efforts, it would be unacceptable to create false hope. Trying to obtain a detailed accounting of the sequence of events would imply that the child's submersion was the mother's fault.
A 50-year-old man presents with a lump in his groin for 2 weeks. He states he was lifting an air conditioner and he felt a pop in his groin and began to notice an outpouching in his lower abdomen that has become mildly tender over the last week. Resting and lying flat appears to help; standing and lifting aggravate it. He denies any fevers, nausea, vomiting, or changes in bowel habits. Patient denies any previous abdominal surgeries or procedures. Upon examination, you identify a soft, reducible mass in the lower abdomen and hernia examination reveals a mass pushing against the side of your finger. You order an ultrasound of the lower abdomen and find the intestinal sac has traversed through a weakened area of the abdominal wall and through Hesselbach's Triangle. Question What type of hernia does this patient have? 1 Direct inguinal hernia 2 Femoral hernia 3 Indirect inguinal hernia 4 Umbilical hernia 5 Incisional hernia
1 Direct inguinal hernia Explanation Direct inguinal hernia is correct because the direct inguinal hernia enters through the weakened abdominal fascia and into the anatomic region known as Hesselbach's Triangle. This area is bordered by the rectus abdominus, the inferior epigastric artery, and the inguinal ligament. Femoral hernias, indirect inguinal hernias, and umbilical hernias do not traverse through Hesselbach's triangle. There was no previous surgery for an incisional hernia to occur. If the hernia strikes the pad or side of the finger, it is usually direct, if it strikes the distal aspect of the finger it is usually indirect.
A 39-year-old woman presents with her sister. The patient is confused, but she is resting comfortably. The sister explains that the patient had a fit and suddenly blacked out. Her body became very rigid and unmovable. Jerking motions of her arms, legs, and head started. They lasted for a few minutes and then stopped. She was unresponsive for about 5 minutes, during which time the sister called the paramedics. Question What is the most likely diagnosis? 1 Grand mal seizure 2 Simple partial seizure 3 Absence seizure 4 Complex partial seizure 5 Status epilepticus
1 Grand mal seizure Explanation The correct answer is grand mal seizures, or tonic-clonic seizures. They are characterized by a tonic phase—which includes a sudden loss of consciousness and rigidity that typically lasts less than a minute—followed by a clonic phase characterized by jerking motions (usually for a few minutes), and then a flaccid coma state. Patients with simple partial seizures do not experience a loss of consciousness. Absence seizures present with an impairment of consciousness (occasionally with mild tonic, clonic, or atonic symptoms), and the patient is typically not aware of the attacks. Complex partial seizures typically have impaired consciousness with focal motor symptoms or somatosensory symptoms, and they do not have the three phases described. Status epilepticus is defined as a fixed and enduring epileptic condition of at least 5 minutes, with repeated seizure episodes and no recovery period between them.
A 65-year-old man presents to the emergency room following initiation of chemotherapy for a recent diagnosis of lymphoma. His past medical history is significant for chronic renal insufficiency. Over the past few days, he has been experiencing vomiting, restlessness, muscle weakness, and tingling sensations. On physical examination, his mucous membranes are dry. His vitals reveal a blood pressure of 90/40 mm Hg and a heart rate of 110/min. Lab results reveal a white blood cell count of 30,000 cells/mm3(normal range: 4,500-11,000 cells/mm3) BUN 38 mg/dL (7-20 mg/dL), Creatinine 2.2 mg/dL (0.6-1.2 mg/dL), potassium 5.0 mEq/L (3.5-5.0 mEq/L ), and calcium of 6.0 mg/dL (8.5-10.2 mg/dL). Question What therapy is the most crucial in managing this patient? 1 IV hydration 2 Hemodialysis 3 Sodium bicarbonate 4 Sodium polystyrene sulfonate 5 Calcium gluconate
1 IV hydration Explanation The correct answer is IV hydration. This patient is exhibiting tumor lysis syndrome. Patients with lymphoma can develop this complication after initiating chemotherapy. The most common presenting signs/symptoms that can develop are restlessness, tachycardia, muscle weakness, increased WBC count, azotemia, hypocalcemia, and hyperkalemia. This patient is exhibiting all of these. In this setting, the most important thing to do is aggressively rehydrate the patient. Hemodialysis is not necessary at this point. Sodium bicarbonate can be used to alkalinize the urine, but the most important step is to rehydrate. Sodium polystyrene sulfonate can be used if the patient is severely hyperkalemic, which this patient is not. Calcium gluconate is usually recommended if the potassium level is above 7 mg/dL.
A 32-year-old man presents with sudden onset of severe chest pain. He describes the sensation as a tearing pain that radiates to his back. He is tall and thin with very long arms and fingers. He has a medical history of lens subluxation and some other eye problems. Based on the patient's history and physical, a dissecting aortic aneurysm secondary to an underlying genetic condition is suspected. Question What bony abnormality is most commonly associated with this patient's condition? 1 Kyphoscoliosis 2 Lordosis 3 Osteoporosis 4 Lordoscoliosis 5 Swayback
1 Kyphoscoliosis Explanation This patient has Marfan syndrome, an inherited autosomal condition. People with Marfan syndrome can be tall, lanky, and thin with long arms and fingers. Ocular involvement is seen with Marfan syndrome, including lens subluxation and myopia. A dissecting aortic aneurysm is a complication of Marfan syndrome. A dissecting aortic aneurysm can present with the sudden onset of severe chest pain, as in the case here. The pain of a dissecting aneurysm is sometimes described as "tearing" and sometimes radiates to the back. Scoliosis and kyphoscoliosis are seen with Marfan syndrome. Kyphoscoliosis consists of both kyphosis and scoliosis. Scoliosis is an abnormal lateral curvature of the spine. Kyphosis is a spinal deformity due to extensive flexion. It is an exaggeration of the thoracic curvature. This produces a "hunchback." Lordosis is an extension deformity. It is an exaggerated lumbar curvature. Lordosis is also called swayback. Osteoporosis is a decrease in the bone mineral density. Lordoscoliosis is a combination of lordosis and scoliosis. It is lateral curvature of the spine and exaggerated lumbar curvature.
A 48-year-old woman has a history of poor interpersonal relationships, chronic headaches, and previous suicide attempts. She has an extremely demanding attitude and often directs her anger towards others. Question What personality disorder is the most likely diagnosis? 1 Narcissistic 2 Avoidant 3 Borderline 4 Histrionic 5 Antisocial
3 Borderline Explanation The clinical picture is suggestive of a borderline personality disorder. These patients typically present with the following clinical findings: they are impulsive, have unstable interpersonal relationships, are suffused with anger and fear, lack self-control, are suicidal, and demonstrate aggressive behavior. Narcissistic personalities are exhibitionists, grandiose, preoccupied with power, and have poor social interactions. Avoidant personalities fear rejection, have low self-esteem, and hyper-react to rejection and failure. Histrionic personalities are dependent, immature, seductive, vain, and egocentric. Antisocial personalities are described as selfish, callous, impulsive, associated with having legal problems, and are promiscuous.
A 76-year-old man presents with substernal chest pain; it is associated with progressive exertional dyspnea, easy fatigability, and dizziness. These symptoms are exacerbated by walking short distances, and they are relieved with rest. He denies fever, chills, cough, wheezing, pleurisy, calf pain, abdominal complaints, peripheral edema, cigarette, drug use, alcohol use, sick contacts, or travel. His physical exam reveals a normal blood pressure and a rough, harsh, low-pitched crescendo-decrescendo systolic murmur beginning after the first heart sound; it is best heard at the second intercostal space in the right upper sternal border. Its intensity is increased toward midsystole; the murmur radiates to both carotid arteries and is accentuated upon squatting, and it is reduced during Valsalva strain. His lungs are without adventitious sounds. Question What medication is most appropriate for the management of the patient? 1 Lopressor (Metoprolol) 2 Vasotec (Enalapril) 3 Nitropress (Nitroprusside) 4 Lasix (Furosemide) 5 Digoxin (Lanoxin)
1 Lopressor (Metoprolol) Explanation This patient's presentation is significant for aortic stenosis. The only definitive treatment for aortic stenosis is aortic valve replacement. The development of symptoms due to aortic stenosis provides a clear indication for replacement. For patients who are not candidates for aortic replacement, percutaneous aortic balloon valvuloplasty may provide some symptom relief. The medical treatment options are limited in symptomatic patients with aortic stenosis who are not candidates for surgery. Beta blockers might be used if the predominant symptom is angina. Those with congestive heart failure or pulmonary congestion may benefit from the cautious use of digitalis, diuretics, and angiotensin-converting enzyme (ACE) inhibitors. Although vasodilators may be used for heart failure and for hypertension associated with aortic stenosis, they should be used with extreme caution to prevent excessive decreases in preload or systemic arterial blood pressure. Reducing the blood pressure to normal levels in patients with hypertension and aortic stenosis is advisable, but hypotension must be avoided.
A 40-year-old woman presents because of a 7-day history of pain in her right arm. The patient denies any trauma or injury to this extremity just prior to the pain starting but does admit to having a Colles' fracture in this arm around 2 months ago. She denies any injury to her back, neck, or other musculoskeletal system prior to the event of pain. She describes the pain as burning and throbbing with an extremely diffuse, uncomfortable aching accompanying it. She further states that this limb has become extremely sensitive to touch and to cold, and it does appear somewhat more swollen than her left arm. The patient is very upset because she does not know why her arm is so painful when she has not done anything to it. She is a non-smoker, does not drink, and exercises 3 times a week. Physical examination conducted of the extremity reveals a slightly cyanotic, mottled right arm with generalized pain of the entire extremity. Pulses are faint (1+) and ROM is limited. You diagnose the patient with Stage I complex regional pain syndrome. Question A multidisciplinary approach is recommended in the treatment of this patient. What pharmaceutical agents is recommended as part of the initial treatment plan? 1 NSAIDs 2 Prazosin 3 Intravenous immunoglobulin 4 Oral calcitonin 5 Epidural clonidine
1 NSAIDs Explanation The patient being described is most likely in Stage I of complex regional pain syndrome (CRPS). This disorder is rare, causing instability of the vasomotor and autonomic components. CRPS is a disorder of a body region, most commonly the extremities, that is characterized by pain, swelling, limited range of motion, vasomotor instability, skin changes, and even patchy bone demineralization. Frequently, symptoms will begin after the patient has sustained an injury, had surgery, or experienced a vascular event, such as a stroke. 35% of patients with CRPS report not having a distinct precipitating event. Stage I of CRPS characteristically has such findings such as pain in the limb (burning/throbbing), diffuse, uncomfortable aching, sensitivity to touch or cold, or even localized edema. The distribution of pain is not compatible with a single peripheral nerve, trunk, or root lesion. There will also be signs of variable vasomotor disturbances that result in altered color and temperature. At this stage, if a radiograph was completed, it will most likely be normal but could show signs of patchy demineralization. All treatment options are found to be much more effective if they are begun as soon as the diagnosis is made or if the patient is diagnosed as Stage I. When considering treatment for this patient, one must take a multidisciplinary approach, including initiation of oral/topical pharmacologic agents, as well as physical and occupational therapy. Less invasive actions should be taken initially, which should include those to help tolerate the pain: NSAIDs, a tricyclic antidepressant, an anticonvulsant, or even an oral opioid. Patients have even been found to have pain relief from bisphosphonates, oral glucocorticoids, or even intranasal calcitonin (the response being more variable, however). Topical treatments, such as capsaicin cream, should also be initiated. The key to success is to use whatever works to reduce the pain to aid the patient's participation in weekly physical therapy. Those who are in Stage I and have no improvement after 2 weeks of therapy or those who are considered Stage 2 or 3 will receive more invasive therapies to help alleviate pain. This will include such actions as epidural clonidine. This treatment may result in significant side effects; hypotension or sedation may be substantial. There are many investigations for treating longstanding or refractory forms of CRPS. These include ketamine infusion, intravenous immunoglobulin administrations, or even mirror visual feedback therapy. These should not be considered first-line treatment regimens.
A 50-year-old man was playing baseball with his company team last weekend and is now experiencing severe pain in his left shoulder. He states that he has been the team pitcher for several years now. He has developed pain progressively in his left shoulder over the last few months. He denies any numbness or tingling in his arm, hand, or fingers of his left upper extremity. Question Which exam would you expect to yield pain? 1 Neer test 2 Tinel sign 3 Phalen test 4 Bragard stretch test 5 Apley grind test
1 Neer test Explanation Neer test is the only exam listed that evaluates for a rotator cuff injury. The Neer test is performed by having the patient internally rotate the shoulder while the practitioner flexes the arm forward. Pain will be elicited as the supraspinatus muscle presses against the acromion. Tinel sign evaluates for any irritated nerve, such as in carpal tunnel syndrome. Phalen test evaluates for carpal tunnel syndrome. Bragard stretch test evaluates for nerve root irritation at the level of L4, L5, and S1. Apley grind test evaluates for a torn meniscus.
A 72-year-old woman presents with a severe unilateral headache, jaw pain, and scalp tenderness. The patient states the headache is of a piercing quality; her jaw hurts only when she chews, and feels better a few minutes after she stops chewing. A pulsation of the temporal artery on the same side as her headache cannot be appreciated, and prednisone is prescribed until the patient can see a specialist. Question What is the reason prednisone is prescribed in this case? 1 Risk for blindness 2 Stenosis of the contralateral temporal artery 3 As treatment for pseudotumor cerebri 4 Edema reduction of berry aneurysm 5 Angioedema in the oropharynx
1 Risk for blindness Explanation The clinical picture is suggestive of giant cell (temporal) arteritis, a systemic arteritis affecting the medium and large blood vessels (arteries) in patients over 50. It is often referred to as temporal arteritis because it frequently involves the extracranial branches of the carotid artery. Signs and symptoms include headache, scalp tenderness, jaw claudication, visual problems, and throat pain. If not treated appropriately, blindness can occur due to the involvement of the posterior ciliary branch of the ophthalmic artery. Prednisone should be initiated immediately. This patient has a vasculitis—not a stenosis—of the artery on the same side as the headache; contralateral is the opposite side. Pseudotumor cerebri is intracranial hypertension with similar symptoms, but treatment is with diuretics and weight loss. A berry aneurysm is an out-pouching of a cerebral blood vessel wall. A typical symptom is described as "the worst headache of my life" by patients. Some symptoms are similar to a temporal arteritis, but treatment is surgical. Angioedema of the oropharynx is usually due to an allergic reaction to numerous agents. Common symptoms are difficulty breathing and throat fullness as a result of airway occlusion. Treatment is the removal of the offending agent and securing the airway (if indicated).
A 32-year-old woman presents with a 3-month history of right hand pain and paresthesia of the thumb, index, and middle fingers. She works in a retail sales office and spends much of the day typing. Question What associated physical finding may indicate an advanced case of this suspected diagnosis? 1 Thenar atrophy 2 Hypothenar atrophy 3 Weakness on extension of the DIP joints 4 Decreased gap to 2 mm on 2-point discrimination test 5 Weak radial artery pulse
1 Thenar atrophy Explanation When compression of the median nerve is severe and long-standing, atrophy of the thenar muscles—which are supplied by the motor branch of the median nerve—can occur, indicating possible denervation of the muscles. Hypothenar atrophy would indicate ulnar nerve long-standing compression, usually because of a space-occupying lesion, such as ganglion, lipoma, ulnar artery aneurysm, or muscle anomaly. Weakness on extension of the DIP joints would indicate injury or entrapment of the radial nerve. Widening of the gap (NOT decrease of the gap) in 2-point discrimination testing would be expected. The radial artery is not within the carpal tunnel and would unlikely be decreased unless there was injury to the wrist.
A 26-year-old woman presents for her second obstetric visit in the first trimester. Routine screening tests (blood typing, testing for syphilis, hepatitis, rubella immunity, and HIV) are performed; the test returns positive for HIV. She is counseled to start antiretroviral therapy and to have a cesarean delivery. Question To reduce the risk of mother-to-newborn transmission, the best drug treatment is a drug that prevents what mechanism of the infection? 1 Virus replication 2 Viral assembly 3 Fusion of virus with the host cell 4 Integration of HIV genetic material into the host chromosome 5 Binding of the HIV virion to the surface of the cells
1 Virus replication Explanation All pregnant HIV-infected patients beyond 14 weeks' gestation should be on a highly active antiretroviral therapy (HART) regimen. With cesarean delivery and appropriate antiretroviral therapy, risk of transmission of HIV to the newborn is less than 1%; the risk would be 25% without such a treatment. Zidovudine is the only anti-HIV drug that is fully approved for use during pregnancy. It is inhibits HIV's reverse transcriptase and is placed within the viral DNA. When placed in the viral DNA by the reverse transcriptase, transcription of the viral genes is inhibited. This prevents virus replication. Other drugs that belong to this group (abacavir, emtricitabine, didanosine, zalcitabine, lamivudine, tenofovir, reverset, and stavudine) are not approved because of side effects. Protease inhibitors (amprenavir, atazanavir, fosamprenavir, ritonavir, indinavir, nelfinavir, saquinavir, and tipranavir) inhibit the retroviral protease from cleaving the viral proteins. HIV produces its own proteases that play role in the production of infective viral particles by cleaving viral proteins to sizes of a mature viral particle (viral assembly). Protease inhibitors inhibit proteases from cleaving the viral proteins; in doing so, they reduce the spread of the virus to other uninfected cells. They are not approved in pregnancy because of their serious side effects. Enfuvirtide interferes with the viral gp41 protein and prevents fusion of HIV with the host cell. It is used in combination therapy with other antiretrovirals when all other treatments have failed. Maraviroc and other CCR5 entry inhibitors act on the human T-cell, changing it in such a way that HIV is unable to bind and therefore continue the replication process. They interfere with the host immune system and cause serious side effects; therefore, they are not approved in pregnancy. Integrase inhibitors, like raltegravir, inhibit the enzyme that integrates HIV genetic material host. Since the integration is a distinct step in HIV life cycle, integrase inhibitors may be taken in combination with other HIV drugs to minimize viral drugs resistance. They are also useful in the case of the resistance to other drugs. Due to their adverse effects, they are not recommended in pregnancy.
A 63-year-old man presents with bleeding gums and a general feeling of malaise. He states that he has been running a low-grade fever for the past few days, feels fatigued, and has bone and joint pain. He thought that he had the flu, but he became concerned when his gums began to bleed. Physical examination reveals a pale individual, mild petechiae on the extremities, gingival hyperplasia, and a palpable spleen. A CBC reveals pancytopenia and blasts with Auer rods on the differential. Question What is the most likely diagnosis? 1 Chronic lymphocytic leukemia 2 Acute myelogenous leukemia 3 Multiple myeloma 4 Chronic myelogenous leukemia 5 Polycythemia vera
2 Acute myelogenous leukemia Explanation The clinical picture is suggestive of acute myelogenous leukemia (AML). AML is a myeloproliferative disorder affecting older individuals; the median age is 60. Most cases arise without any clear cause, but many chemical agents and radiation are linked to AML. Features of this disorder include short duration of fever, fatigue, and bleeding. The CBC will reveal pancytopenia or cytopenia. Blasts can be seen in the peripheral smear in 90% of patients. Cytoplasmic inclusion bodies (Auer rods) are pathognomonic of this disorder. Chronic lymphocytic leukemia is characterized by isolated lymphocytosis. Multiple myeloma is a malignancy of plasma cells. Chronic myelogenous leukemia is characterized by a very high WBC count (>100,000/mcL). Polycythemia vera is characterized by overproduction of all three hematopoietic cell lines, primarily the RBC cell line.
A 4-year-old boy presents with a 1-month history of a red area on his cheek that has gradually enlarged. The rash is slightly pruritic but is nontender. The patient had an upper respiratory infection 3 weeks ago that lasted 5 days. He had a low-grade subjective fever for the first 2 days of that illness. There is no recent travel history. The patient was given a new kitten for his birthday 2 months ago. Past medical history and family history are unremarkable. On examination, the patient is well-appearing and afebrile. He has a 3-cm annular erythematous lesion on his right cheek. It is slightly scaly with central clearing and a papulovesicular border. The rash is nontender to touch. There is no cervical adenopathy. Eyes, ears, and pharynx are normal to examination. Wood lamp examination reveals no fluorescence. Question What is the most appropriate treatment for this patient? 1 Hydrocortisone 1% ointment topically tid for 2 weeks 2 Clotrimazole 1% cream topically tid for 3 weeks 3 Nystatin cream 100,000 units/g topically tid for 3 weeks 4 Amoxicillin 500 mg po tid for 7 days 5 Cephalexin 250 mg po tid for 7 days
2 Clotrimazole 1% cream topically tid for 3 weeks Explanation The correct response is clotrimazole 1% cream topically tid for 3 weeks. Tinea corporis (ringworm) is a common skin lesion found in children. It is caused by superficial infection of glabrous (non-hairy) skin with a variety of fungi. In children, the causative agent is usually Microsporum canis, Microsporum audouini, or Trichophyton mentagrophytes. Adults are more likely to be infected with Trichophyton tonsurans, Trichophyton mentagrophytes, Trichophyton rubra, or Trichophyton verrucosum. The lesion has a characteristic appearance that is annular and scaly with a well-demarcated vesicular, pustular, or papular border. If the lesion has been present for any significant length of time, there will be central clearing of the lesion. The infection is often contracted from young kittens or puppies. Dermatophyte infections of the hair with Microsporum audouinii and canis produce a brilliant green fluorescence under Wood lamp, while Trichophyton schoenleinii produces a pale green fluorescence. Trichophyton tonsurans and violaceum do not fluoresce. It should be noted that only the hair fluoresces in the case of dermatophyte infection, not the skin. Cellulitis due to Streptococcus pyogenes or Staphylococcus aureus is characterized by erythema, induration, and tenderness of the skin. It often follows some trauma to the skin. The infection may be superficial in the case of erysipelas, caused by group A streptococcus; it also may be deeper, involving subcutaneous skin. The skin surface is usually smooth, although Staphylococcus aureus may cause exfoliation, which is seen in toxic epidermal necrolysis due to an exfoliative toxin. Candidal skin infection (moniliasis) has a predilection for intertriginous areas, and it is uncommon on the face. It has a macerated, shiny red appearance with small "satellite" pustules at the border. Nummular eczema is a discoid, or coin-shaped, plaque caused by dry skin and possible atopy. There is no central clearing or "active" border to the lesion. Localized dermatophyte infections may be treated topically with clotrimazole. Griseofulvin is associated with liver toxicity and aplastic anemia, and it should be reserved for dermatophyte infections of the hair (tinea capitis and tinea barbae) and nails (onychomycosis) or chronic, persistent infection of the feet (tinea pedis). Nystatin is an effective topical treatment for candidal infections, but it is ineffective in treating dermatophytosis. Topical steroids are an appropriate treatment for nummular eczema, but they only mask the clinical features of tinea corporis (tinea incognito) without treating the infection. Amoxicillin and cephalexin are appropriate therapy for uncomplicated cellulitis due to Streptococcus pyogenes and Staphylococcus aureus, respectively.
A 3-year-old boy presents with difficulty in breathing and a cough that sounds like a seal. On examination, the child has fever, a harsh barking cough, a respiratory rate of 38/minute, and minimal stridor on agitation. On lung auscultation, there are no rales or wheezing. On cardiac auscultation, there is tachycardia. Radiological examination reveals steeple sign. Question What is the most likely diagnosis? 1 Bronchiolitis 2 Croup 3 Epiglottitis 4 Foreign body aspiration 5 Pneumonia
2 Croup Explanation Croup, caused by the parainfluenza virus, is an upper respiratory infection with a characteristic brassy cough, hoarseness, and respiratory stridor. The typical clinical picture has a variable onset, from a progressive fever, sore throat, and cough for a couple of days to a sudden midnight onset. The severity varies from stridor only (with mild agitation) to stridor at rest to airway obstruction. Treatment involves humidified air, inhaled or oral corticosteroids, and nebulized racemic epinephrine. Bronchiolitis, caused by the respiratory syncytial virus (RSV) and other respiratory viruses (influenza, parainfluenza, rhinovirus), is a severe infection in infants that can progress to respiratory failure. Premature infants and infants with heart and lung disease have severe forms of bronchiolitis. Patients have wheezing, rales, prolonged expiratory phase, rhinorrhea, nasal congestion, fever, tachypnea, and respiratory distress. Antigen testing for RSV from nasal secretions provides a rapid diagnosis. Ribavirin administered through aerosol may shorten the clinical course. Polyclonal and monoclonal antibodies are used for prophylaxis in winter. Epiglottitis is characterized by inflammation and rapidly progressive edema of the epiglottis and contiguous tissue. Children ages 2-7 who missed some immunizations are prone to this infection during winter. Drooling, hoarseness, high fever, sore throat, the characteristic "sniffing dog" position, and the rapidly progressive respiratory obstruction make the diagnosis a clinical one. If radiographs are taken, they may show the "thumb sign" on lateral films caused by an edematous epiglottis, but radiographs should not delay airway management and are not necessary for diagnosis. Immediate intubation and intravenous ceftriaxone or cefuroxime are recommended for treatment. Foreign body aspiration presents with complete or partial airway obstruction localized at a different location in the respiratory tract. The acute presentation after a choking episode provides the diagnosis, but without a witnessed episode of choking, clinical presentation may take a couple of days after the aspiration. Tracheal obstructions present with acute asphyxia in total obstruction and with stridor in partial obstruction. Localized signs, such as wheezing, rhonchi, and decreased breath sounds, are found in lower respiratory tract obstruction. The treatment of choice is extraction with a rigid bronchoscope. Pneumonia in children may be a difficult clinical diagnosis. The immune status, the age, and risk factors may help in suspicion of the etiologic organism. Profuse crackles and rales after an upper respiratory infection may suggest a viral pneumonia. A localized consolidation syndrome is more characteristic for a bacterial origin of pneumonia. The characteristic findings on chest radiography also suggest the etiology of the pneumonia.
A 42-year-old hypertensive, diabetic Native American woman is scheduled for a follow-up visit today. She says that her father recently passed away after having a heart attack. She is extremely worried and wants to know what she can do to reduce her risk of cardiovascular disease (CAD). You explain to her that the risk factors for CAD are classified as either non-modifiable or modifiable. Question What is the strongest non-modifiable risk factor for CAD in this woman? 1 Age 2 Ethnicity 3 Hypertension 4 Diabetes 5 Sex
2 Ethnicity Explanation Native Americans are at a relatively high risk of CAD. The risk factors for CAD may be classified as non-modifiable and modifiable. The non-modifiable risk factors include increasing age (>45 for men and >55 for women), male sex, positive family history of CAD, and ethnicity. African Americans, Native Americans, Mexican Americans, and some Asian groups are at a greater risk than others, which is partly due to higher incidences of hypertension, diabetes, and obesity among these groups. The modifiable risk factors include diabetes, hypertension, smoking, body weight, and blood lipid levels. A sedentary lifestyle and fat- or salt-rich diets are also considered risk factors.
A 60-year-old man with a history of recurrent sinus infections presents with hemoptysis and hematuria. Physical examination shows a temperature of 101°F, a blood pressure of 145/85 mm Hg, decreased breath sounds on his right lower lobe, and palpable purpura on his bilateral lower legs. No warm or swollen joints were noted. Anti-neutrophil cytoplasmic antibody (ANCA) Positive Anti-double-stranded DNA (dsDNA) Negative Anti-Glomerular Basement Antibody (GBM) Negative Erythrocyte sedimentation rate 50 mm/hour Hemoglobin 11 g/dL Serum creatinine 1.5 mg/dL Serum potassium 5 meq/L Serum chloride 21 meq/L Urinalysis 1.020, pH 6, numerous red cells, 500 mg albumin/g creatinine, red blood cell casts Chest X-ray Right lower lobe effusion Question What is the most likely diagnosis? 1 Systemic lupus erythematosus (SLE) 2 Granulomatosis with polyangitis (GPA, Wegener's granulomatosis) 3 Rheumatoid arthritis (RA) 4 Anti-GBM antibody disease (Goodpasture's disease) 5 Immunoglobulin A vasculitis (IgA vasculitis, Henoch-Schönlein Purpura, HSP)
2 Granulomatosis with polyangitis (GPA, Wegener's granulomatosis) Explanation This patient most likely has granulomatosis with polyangiitis, formerly known as Wegener's granulomatosis. While many of the answer choices can have palpable purpura, hemoptysis, hematuria, and signs of glomerular disease, granulomatosis with polyangiitis is most likely to have this combination of symptoms and lab results. Lack of arthritis makes RA and SLE much less likely. Lack of anti-dsDNA further reduces the likelihood of SLE. Patient's adult age makes IgA vasculitis/Henoch-Schönlein Purpura less likely. Lack of anti-GBM antibodies rules out anti-GBM antibody disease (Goodpasture's disease). Granulomatosis with polyangiitis is a small vessel vasculitis of unknown pathogenesis that causes inflammation and necrotizing granulomas in the upper and lower respiratory tract and kidneys, as well as rash; 15-50% of patients have cutaneous involvement. Palpable reddish-purple lesions on the lower extremities should raise suspicion of vasculitis; macules and nodules may also erupt. 50-95% of patients with this disease will eventually have renal involvement, which may range from mild findings to fulminant dialysis-requiring nephritis. On renal biopsy, focal segmental necrotizing glomerulonephritis is noted; crescents may be seen. Immune staining is generally negative. Treatment involves immunosuppressive therapy, as is the case with SLE, and dialysis as needed. In some resistant cases, plasmapheresis may be helpful. Patients with granulomatosis with polyangiitis/Wegener's are most commonly anti-neutrophil cytoplasmic antibody (ANCA)-positive (90% of patients). Patients with anti-GBM antibody disease (Goodpasture's disease) typically present with a triad of glomerulonephritis, pulmonary hemorrhage, and anti-glomerular basement membrane (GBM), type IV collagen antibodies. 10-40% of patients with anti-GBM antibody/Goodpasture's disease are ANCA-positive at time of diagnosis, but negative anti-GBM antibody makes this diagnosis a much less likely answer. Palpable purpura is not typically a symptom of anti-GBM antibody disease. Systemic complaints and signs such as malaise, weight loss, fever (as with this patient), or arthralgia, are also typically absent. Systemic symptoms should prompt consideration of a concurrent vasculitis. Systemic lupus erythematosus (SLE) is a systemic inflammatory disease that causes a variety of signs and symptoms affecting many organs. Renal involvement is frequent in SLE and may include a variety of glomerular diseases, causing hematuria, proteinuria, and urinary casts. Though ANCA can be positive in about 31% of patients with SLE, it would be less likely to be anti-double-stranded DNA-negative. Anti-dsDNA antibodies are seen in approximately 70% of patients with SLE. Also, the patient in the question has no joint symptoms; 95% of SLE patients have arthritis. Palpable purpura and pulmonary hemorrhage can sometimes be present in SLE, as they are in this patient. Immunoglobulin A vasculitis, formerly known as Henoch-Schönlein Purpura (HSP), is a vasculitic syndrome that is seen more often in children than adults; it may cause petechiae, palpable purpura, gastrointestinal bleeding, and hematuria/glomerulonephritis. Complement counts and anti-double-stranded DNA are normal, and elevated IgA antibody levels may be seen. Severe lung involvement, such as pulmonary hemorrhage (hemoptysis), is rare in patients with IgAV (HSP). Lack of arthritis makes this answer choice less likely since arthritis/arthralgia occur in up to 84% of patients with IgA vasculitis. While it can occur rarely in adults, it is primarily a childhood disease that occurs between the ages of 3 and 15. Rheumatoid arthritis (RA) is an inflammatory arthritis that causes joint stiffness and swelling that takes a long time to loosen up after prolonged sitting or sleeping. Symmetrically swollen joints and rheumatoid factor may be noted. While palpable purpura can occur in RA, vasculitis is uncommon (Harrison's chapter 314). RA patients can also have pulmonary hemorrhage with severe respiratory manifestations of RA. ANCA can occur in virtually all inflammatory rheumatologic disease, but lack of joint swelling or pain makes RA a much less likely answer.
A 12-year-old boy presents with fatigue and jaundice. History obtained from the patient and his mother is negative for recent illness, fever, infectious exposures, medication, alcohol, or drug use. He denies gastrointestinal (GI) symptoms and a history of GI disease. On physical examination, he appears ill; the liver edge is palpable and slightly tender. Skin and sclera are icteric, and there is corneal discoloration. On eye examination using a slit-lamp, you note brown-yellow rings encircling the iris in the rim of the cornea bilaterally. You order a serum ceruloplasmin level, which is reported as low. Question What is this diagnostic corneal pigmentation known as? 1 Fleischer's rings 2 Kayser-Fleischer rings 3 Rust rings 4 Arcus juvenilis 5 Pinguecula
2 Kayser-Fleischer rings Explanation Kayser-Fleischer rings are the result of the accumulation of copper in the cornea; they are the most unique sign of Wilson's disease. Wilson's disease is an inherited disorder of copper toxicity due to a genetic defect in copper transport. Beginning at birth, copper is not secreted into the bile or incorporated into the copper protein ceruloplasmin, resulting in low serum levels of ceruloplasmin. Symptoms and signs develop age 5-40 as copper accumulates in the liver, brain, cornea, kidney, and reproductive organs. 50% of patients present with hepatitis, 40% present with neurological manifestations (tremor, speech disorders, dysphagia, incoordination), and 5-10% first present with Kayser-Fleischer rings, amenorrhea, miscarriages, or hematuria. Diagnosis is confirmed by Kayser-Fleischer rings on slit lamp examination in the presence of a low serum ceruloplasmin. AST and ALT levels are often elevated; serum copper is low; 24-hour urinary copper excretion is elevated. Treatment is lifelong chelation or oral zinc and a low copper diet. Keratoconus is a bulging of the cornea to form a cone, and the classic sign is Fleischer's rings, which are iron-colored rings surrounding the cone. This progressive bulge is due to a weakness in the cornea and often occurs bilaterally at age 10-20. There are frequent changes in visual acuity, necessitating repeated prescription changes; contacts provide better correction than glasses. Corneal transplant may be necessary if corrective lenses are not adequate. Arcus juvenilis is a gray or white arc around the peripheral cornea similar to arcus senilis in adults. It occurs in younger adults and is often associated with high blood cholesterol. A metallic foreign body lodged in the cornea can quickly result in a single small-diameter rust ring that requires ophthalmologic intervention with a rust ring drill for removal. A pinguecula is a raised, yellowish discoloration on the bulbar conjunctiva at the 3 o'clock or 9 o'clock position of the scleral-corneal junction. It is a benign growth caused by an accumulation of conjunctival tissue that can be the result of chronic actinic irritation.
A 20-year-old man has had a 1-month duration second-line acne therapy; he now presents with skin that looks worse and has more inflammation and cystic development. This patient has now progressed to severe acne. Question What is the most appropriate next step? 1 Stop all medications and review in 2 weeks. 2 Perform skin biopsies. 3 Perform blood cultures. 4 Add oral isotretinoin after stopping medication for 2 weeks. 5 Add emollient cream and a sulfur-based cleanser.
4 Add oral isotretinoin after stopping medication for 2 weeks. Explanation Systemic treatment with isotretinoin is indicated for patients with cystic acne or with acne that has not responded to conventional therapy. Systemic retinoids are teratogenic and should be avoided in pregnant women or women planning to conceive. Informed consent must be obtained before its use, and patients must be enrolled in a monitoring program (iPledge). For severe acne, the medications are continued, not discontinued. Skin biopsies and blood cultures are not indicated for acne treatment. Emollient creams are used to soften skin. Sulfur-based cleansers are indicated for treating acne but are not indicated for inflammatory or cystic-type acne.
A 19-year-old woman presents with worsening headaches. She reports a multi-year history of episodic, throbbing headaches. They have intensified, and she now misses classes and work periodically due to her headaches. The headaches occur about 4-6 times per month recently, up from 1-2 per month when she first started experiencing headaches. Her headaches last 2-3 days and are accompanied by nausea, vomiting, and light sensitivity. After the headache resolves, she denies any residual symptoms. She denies neurologic symptoms, such as vision or taste changes, gait disturbances, and memory loss. She has tried multiple over-the-counter pain medications without relief. Her mother and maternal aunt experienced similar headaches. She reports some increased stressors and less sleep since recently starting college. Her past medical history is unremarkable, with no known medical conditions (except for the headaches), no surgeries, no chronic medications, and no drug allergies. She has never been sexually active and reports regular menses. She denies the use of tobacco, alcohol, and drugs. On physical exam, the patient appears comfortable and reports no headache at this time. Her entire exam, including neurological, is normal. Question If testing was performed on this patient for her headaches, which of the following test results best support this patient's likely diagnosis? 1 Inflammation on temporal artery biopsy 2 Normal magnetic resonance imaging (MRI) 3 Polyspikes and slow wave pattern on electroencephalogram (EEG) 4 Positive toxicology for cocaine 5 Thickened sinus mucosa on computed tomography (CT)
2 Normal magnetic resonance imaging (MRI) Explanation This patient is suffering from migraine headaches, common headaches classically described as throbbing in nature and accompanied by nausea (and possibly vomiting), photophobia (sensitivity to light), phonophobia (sensitivity to sounds), and possibly visual changes and aura. Migraines are more common in women than men. They are usually a chronic condition, beginning in adolescence or early adult years. Often, a family history is present. Stress, sleep deprivation, and various foods can be among many triggers for migraines. With a classic history and no alarm features of a more serious condition, no testing would be necessary. If testing was performed, this patient would be expected to have normal magnetic resonance imaging (MRI). Inflammation on temporal artery biopsy indicates temporal arteritis, which can cause a throbbing headache. Temporal arteritis is more likely to occur in an older person and is not associated with nausea and vomiting, nor does it occur episodically in nature like migraines. Patients with temporal arteritis may experience some visual symptoms, such as amaurosis fugax, diplopia, and vision loss. Polyspikes and slow wave pattern on electroencephalogram (EEG) indicate seizure activity. This patient's symptoms and headache pattern are not associated with any seizure disorder. If the patient had positive toxicology for cocaine, she could present with a nondescript headache and nausea or abdominal pain related to the vasoconstrictive properties of cocaine. In acute cocaine use, restlessness and mydriasis (dilation of the pupils bilaterally) may be observed. Nothing in this patient's history or physical suggests cocaine use as the cause of her headaches. Thickened sinus mucosa on computed tomography (CT) indicates chronic sinusitis, which can cause headaches. The headaches associated with chronic sinusitis can occur episodically but would likely be accompanied by nasal congestion, thick and/or discolored nasal discharge, sinus pressure, and possibly post-nasal drip with sore throat. Sinusitis headaches would not be expected to produce the episodic pattern described with this patient's migraines.
A 45-year-old man is admitted to the medical floor of a hospital with increasing jaundice, swollen legs, and episodes of disorientation for the last several weeks. His abdomen is distended and he looks ill. According to the family, he is an alcoholic and has recently been diagnosed with cirrhosis of liver. On examination, he has a temperature of 99°F, BP 100/72 mm Hg, and his pulse is 86/minute. Sclerae are icteric, and he has 2+ pitting pedal edema. Lungs are clear, and heart sounds are normal. Abdomen is distended with moderate ascites, caput medusae, and no tenderness. Liver and spleen are not palpable. He is awake but drowsy and oriented to person and place but not to time. He does have a fine tremor in his hands. Labs show WBC 8000/uL, platelets 100,000/uL, Hb 12g%, AST 76 U/L, ALT 56 U/L, AP 62 U/L, and ammonia is 124 mg/dL. Question In order to avoid full decompensation into hepatic encephalopathy, what is the most important action to take at this time? 1 Increase dietary protein. 2 Prevent constipation. 3 Sedate the patient to avoid injury from disorientation. 4 Start IV antibiotics empirically anticipating infection. 5 Add thiamine and folic acid for nutritional support.
2 Prevent constipation Explanation This patient is already in the initial phases of hepatic encephalopathy due to alcoholic liver disease, as evidenced by the disorientation and tremors, also known as asterixis. The primary cause of hepatic encephalopathy is unclear. Metabolic abnormalities due to liver dysfunction—resulting in a spectrum of neuropsychiatric signs and symptoms—are seen. High levels of ammonia are found in the blood. Constipation causes increased ammonia production and absorption due to prolonged intestinal contact and aggravates the condition.Lactulose should be administered frequently to eliminate the ammonia in the stool. It is an indigestible sugar that acts as an osmotic laxative by increasing the water content of the stool and promoting bowel movements. It is digested by the colonic bacteria, and the acidic remains convert ammonia into ammonium ions in the colon, which Lactulose should be administered frequently to eliminate the ammonia in the stool. It is an indigestible sugar that acts as an osmotic laxative by increasing the water content of the stool and promoting bowel movements. It is digested by the colonic bacteria, and the acidic remains convert ammonia into ammonium ions in the colon, which are then excreted in the stool. Oral antibiotics can lower blood ammonia levels by decreasing ammonia production and absorption. The commonly used ones are neomycin, metronidazole, vancomycin, and, lately, rifaximin. The last 3 are better tolerated than neomycin. However, antibiotics have their side effects and can cause bacterial overgrowth syndromes. Their main use continues to be in patients who cannot tolerate disaccharides like lactulose. Acarbose and fermentable fiber can also cause decrease in intestinal ammonia production and absorption. Newer studies with sodium benzoate are ongoing. Benzoate and glycine react to form hippurate, and for every mole of benzoate utilized this way, one mole of nitrogen is excreted in the urine, thereby enhancing ammonia metabolism. This, however, still needs to be studied further to be used widely. Studies with ornithine-aspartate are also being done as a stimulator of ammonia metabolism.All the products mentioned are yet to replace lactulose as the first line of treatment but are potentially useful once more studies are done. Other precipitating factors for hepatic encephalopathy include azotemia, hypokalemia, gastrointestinal bleeding, high protein diet, alkalosis, infection, sedatives, and other hepatotoxic agents. All the products mentioned are yet to replace lactulose as the first line of treatment but are potentially useful once more studies are done. Other precipitating factors for hepatic encephalopathy include azotemia, hypokalemia, gastrointestinal bleeding, high protein diet, alkalosis, infection, sedatives, and other hepatotoxic agents. High protein diet is a contraindication in this condition, as protein catabolism causes increase in ammonia levels. Daily protein should be restricted to 40 g/day. Sedation of the patient should also be avoided since sedatives cause cerebral depression and worsening of encephalopathy. These drugs are also not metabolized adequately by the diseased liver. Though infection is an important precipitating factor for hepatic encephalopathy, empirictreatment is not recommended. However, early and adequate treatment of an infection should be done, especially for spontaneous bacterial peritonitis. In fact, multiple randomized control trials have been done regarding antibiotic prophylaxis for SBP, and they have shown not only a decrease in bacterial infections but also a significant reduction in mortality. Prophylaxis is recommended in cirrhotic patients with risk factors for SBP like GI bleeding, prior history of SBP, and low ascitic fluid protein. Thiamine and folic acid should be added for nutritional support for all alcoholic patients since they are malnourished and vitamin-depleted. However, this will not change the outcome in hepatic encephalopathy as quickly or as much as avoiding constipation.
A 56-year-old man wakes up in the morning to find that he has a swollen, red, and painful big toe on his left foot. He had been on a cruise to the Bahamas 2 days earlier, and he spent much of the time eating and drinking. He normally has 1 glass of wine with dinner on the weekends, but his alcohol consumption increased substantially while on the cruise. He also did a great deal of walking in an attempt to make up for his excesses. He goes into his physician's office, and tests are run. An X-ray shows no acute fracture, and his vital signs are within normal limits. Blood work shows an increase in uric acid, but it is otherwise normal. He begins treatment and feels better within 24 hours. Question What most accurately describes the typical presentation of this disease? 1 Polyarticular arthritis 2 Small lower extremity joints 3 Caucasians more likely than African Americans 4 Women more likely than men 5 Under age 25
2 Small lower extremity joints Explanation Gout presents as acute monoarticular arthritis in 90% of patients. In early gout, usually only 1 or 2 joints are involved. Usually, they are the smaller lower extremity joints. Podagra, or inflammation of the first metatarsophalangeal joint, is the initial joint manifestation involved in about half of all cases, but podagra may also be observed in patients with pseudogout, reactive arthritis, gonococcal arthritis, psoriatic arthritis, and sarcoidosis. The attacks usually begin abruptly and can reach maximum intensity in 6-12 hours. The joints are red, hot, and extremely tender. Untreated, the characteristics of gout change over time. The attacks become more polyarticular. Although more joints may become involved, inflammation in a given joint may become less intense. Attacks occur more frequently and last longer. Eventually, patients may develop a chronic polyarticular arthritis, which can be symmetrical and resemble rheumatoid arthritis. Tophi, which are collections of uric acid crystals in the soft tissues, occur frequently in untreated patients. They can be found in multiple locations, including the fingers, toes, in the olecranon bursae, and along the olecranon (where they may appear to be rheumatoid nodules). Tophi tend to develop after 10 years in untreated patients who develop chronic gouty arthritis. African American patients are usually affected more than Caucasian ones. Symptoms occur more commonly in men than in women. They usually present in the age group 30-50. In women, it occurs more commonly in the postmenopausal age group.
A 31-year-old female nurse who works at a local hospital presents with a purpuric rash covering her arms, legs, and abdomen, as well as fever, chills, nausea, abdominal tenderness, tachycardia, and generalized myalgias. Prior to the development of the rash, the patient noted that she had a headache, cough, and sore throat. Laboratory studies were positive for gram-negative diplococci in the blood, along with thrombocytopenia and an elevation in PMNs. Urinalysis showed blood, protein, and casts. Vital signs are as follows: BP 92/66, P 96, RR 14, T 39. The patient denies any foreign travel and does not have any sick contacts. She is admitted to the hospital and placed in respiratory isolation. Question What major course of therapy should this patient receive? 1 Steroids 2 Supportive care 3 Antibiotics 4 Transfusion 5 Bactericidal/permeability-increasing protein
3 Antibiotics Explanation Antibiotics are the treatment of choice for meningococcemia. The preferred drug for active infection is a third generation cephalosporin. Patients will also receive supportive care, but antibiotic therapy must be initiated quickly if the patient is to survive. Intensive care placement may be necessary if organ failure is imminent. Ventilatory support, inotropic support, and IV fluids are necessary in some. If adrenal insufficiency occurs, corticosteroid replacement may be considered. A central venous line helps to provide large amounts of volume expanders and inotropic medications for adequate tissue perfusion. Steroids have not been shown to play a major role in the treatment of meningococcemia. However, they have been used in addition to antibiotic therapy. In the case of adrenal insufficiency, for example, steroid replacement has been shown to be beneficial. Transfusion does not generally play a major role in treatment. If the patient suffers from a devastating coagulopathy, blood or blood products may be replaced as necessary. Bactericidal/permeability-increasing protein is a protein stored in the granules of neutrophils. It binds to endotoxin in vitro and neutralizes it. This technique is experimental, and it is not used in everyday treatment of meningococcemia.
A 61-year-old man with known cirrhosis presents with a 1-week history of "puffy" ankles and increased shortness of breath. A week prior to symptom onset, he had traveled on vacation, where he engaged in walking, sightseeing, and eating out. He admits more dyspnea with lying down and with increased exertion. His shoes feel snug, and he notes a definite line from wearing socks. The patient denies chest pain, leg pain, fevers, claudication, nausea, headache, lethargy, and hemoptysis. His past medical history is remarkable for cirrhosis and a history of alcoholism. He is awaiting a liver transplant. He had a liver biopsy, but no other surgeries. He takes no medications, has no allergies, and has been abstinent of alcohol for 9 months. He lives with his wife, works as an electrician, and smokes 1 pack of cigarettes per day. Vitals are normal, including oxygen saturation. On physical exam, the patient appears in no acute distress and with normal mental status. His physical exam is remarkable for mild jugular venous distention, 2-3+ edema in lower extremities, and mild dullness to lung percussion. No hepatomegaly or ascites is noted. This patient's laboratory results are shown in the chart. Complete blood count (CBC) Normal Chest X-ray Mild bilateral effusions Urinalysis Normal Beta-natriuretic peptide (BNP) Normal Complete Metabolic Panel (CMP) Abnormal as indicated below Test Result Normal range Units Sodium 126 134-144 mmol/L Potassium 3.4 3.4-4.9 mmol/L Chloride 100 100-109 mmol/L HCO3 26 20-31 mmol/L Glucose 98 70-99 mg/dL Bun 18 7-18 mg/dL Creatinine 1.1 0.6-1.2 mg/dL Calcium 8.8 8.8-10.5 mg/dL Albumin 2.6 3.5-5.0 g/dL Total Protein 5.9 6.4-8.2 g/dL AST (SGOT) 112 15-37 U/L ALT (SGPT) 108 5-43 U/L Alk Phosphatase 158 50-136 U/L Total Bilirubin 1.9 0.1-1.2 mg/dL eGFR >60 >60 mL/min/1.73 m2 Osmolality 265 285-293 mmol/kg H2O Question What is the most appropriate intervention for this patient's fluid and sodium status? 1 Administer intravenous hypertonic saline solution. 2 Advise the patient to contact his gastroenterologist for an appointment. 3 Begin intravenous or oral diuresis. 4 Evaluate for possible pulmonary embolus. 5 Initiate hemodialysis.
3 Begin intravenous or oral diuresis. Explanation This patient is in a hypervolemic hypotonic hyponatremic state. The hypervolemia can be identified primarily by history (edema and dyspnea) and physical exam (jugular venous distention, edema, and pulmonary effusion). The hypotonia (low serum osmolality) and hyponatremia most likely are due to a "dilution" effect, in which this patient's overall fluid status is high, but the sodium levels are normal or even elevated. This patient has hypoalbuminemia (common with chronic liver disease) and increased extracellular fluid, which is causing his mild pulmonary effusions and dyspnea. The most appropriate treatment for this patient is to begin intravenous or oral diuresis. Diuretics should be used with caution in patients with cirrhosis, but his pulmonary edema must be addressed. It would be potentially harmful to this patient to administer intravenous hypertonic saline solution, especially as his overall sodium level is likely normal to high. Hypertonic saline should only be used in acutely ill patients, with careful monitoring. Without prior records, it is difficult to assess any potential worsening in this patient's chronic cirrhosis. While it would be acceptable to advise the patient to contact his gastroenterologist for an appointment in addition to other interventions, it would not address his current hypervolemic hyponatremia or his pulmonary edema. With a complaint of shortness of breath, especially in a smoker, it could be rationalized to evaluate for a possible pulmonary embolus. However, he does not meet the Wells criteria for a PE (no suspected DVT, no tachycardia, no immobilization, no history of DVT/PE or malignancy, no hemoptysis, and alternative diagnoses are more likely than PE). Further evaluation for PE is not indicated. With severe electrolyte abnormalities, or those that do not respond to diuresis, it would be reasonable to initiate hemodialysis. This patient's abnormalities are not severe, and he is mildly symptomatic. A trial of diuresis is indicated before considering hemodialysis.
A 24-year-old man undergoes a routine medical check-up to become a volunteer in the ER. PPD skin test shows an induration diameter of 9 mm. Sputum and chest X-ray are done; they are negative for tuberculosis. He is otherwise healthy and has no fever, cough, or other symptoms of Mycobacterium tuberculosisinfection. Question: What is the most appropriate explanation for the doubtful tuberculin test in this patient? 1 Sputum and chest X-ray report could be wrong. 2 Adequate sputum samples were not supplied. 3 He was given BCG vaccination earlier in life. 4 He may suffer from chronic cavitary lesion in the lung. 5 He has an active tubercular infection.
3 He was given BCG vaccination earlier in life. Explanation The patient was given BCG vaccination earlier in life. The Mantoux tuberculin skin test (TST) is performed to determine whether the person is infected with Mycobacterium tuberculosis. The TST is performed by injecting 0.1 mL of tuberculin purified protein derivative (PPD) into the inner surface of the forearm. Skin test results should be read 48-72 hours after administration of the PPD. Generally, in immunocompetent individuals, any palpable induration ≥10 mm is considered a positive reaction. In the case of tuberculosis suspects or close contacts of individuals with tuberculosis, an induration ≥5 mm should be interpreted as a positive reaction. An induration measuring 5-9 mm is considered a doubtful reaction. In case of doubtful reaction, the possibility of skin sensitivity due to previous immunization or atypical mycobacterium exposure should be considered. Though prior BCG vaccination increases the risk of a reactive PPD, this effect is known to be inconsistent. Studies have shown that reactions >10 mm should not be attributed to prior BCG vaccination. Induration ≥5 mm should be considered positive in HIV-positive patients, patients with known contact with active TB, patients with a history of organ transplant on immunosuppressive therapy, and patients with fibrotic lesions on chest X-ray. Positive tuberculin test indicates exposure of the immune system to tuberculous protein, either in the form of BCG vaccine, an active tuberculous infection, or a chronic tuberculous infection. Proper vaccination history could have unmasked the cause of the indeterminant reaction to tuberculin test in this patient. Other causes, such as a wrong sputum and chest X-ray report, inadequate sputum samples, chronic cavitary lesion, or active tubercular infection, are not the cause of doubtful reaction in this patient.
A 50-year-old man presents with a 2-week history of not being able to see well. He is not on any medications. He has been smoking 2 packs of cigarettes a day for the past 30 years. On examination of his right eye, there is ptosis and miosis. A chest radiograph reveals a rounded opacity in the right lung field. Question What is the most likely diagnosis? 1 Lambert-Eaton syndrome 2 Superior vena cava syndrome 3 Horner syndrome 4 Cushing syndrome 5 Syndrome of inappropriate antidiuretic hormone secretion
3 Horner syndrome Explanation Horner syndrome is due to a bronchogenic carcinoma tumor mass extending to the sympathetic chain. Horner syndrome is comprised of miosis, ptosis, enophthalmos, and hemianhidrosis. Patients may also have atrophy of hand muscles. Lambert-Eaton syndrome is an immune-mediated disorder of neuromuscular transmission resulting in impaired release of acetylcholine from nerve terminals. Patients complain of weakness of the scapular and pelvic girdle muscles and the resultant difficulties in performing activities such as climbing stairs. Other symptoms (e.g., dry mouth, paresthesias, and sexual impotence due to autonomic dysfunction) may also be present. On examination, there is proximal muscle weakness. Deep tendon reflexes are decreased or absent. Superior vena cava syndrome is obstruction of the superior vena cava with supraclavicular venous engorgement. Cushing syndrome may develop as a result of the small cell carcinomas secreting ectopic adrenocorticotropic hormone. Patients may complain of bruising easily. On examination, they may have red-purplish striae, especially over their abdomen, in addition to the typical moon facies and truncal obesity. They may also have muscle wasting, especially of the proximal limb girdle muscles. Laboratory investigations reveal hypokalemia, high plasma ACTH, as well as increased serum and urine cortisol. The syndrome of inappropriate antidiuretic hormone secretion is a paraneoplastic syndrome associated with bronchogenic carcinoma. There is excessive water reabsorption by the kidneys due to the elevated antidiuretic hormone. Patients may complain of feeling weak and lethargic; other symptoms include confusion and coma. Laboratory tests reveal hyponatremia, high urine sodium, low serum osmolality, and high urine osmolality in a euvolemic patient.
A 32-year-old woman presents with paroxysmal episodes of headache, palpitations, and diaphoresis; the episodes last 20 minutes. During one episode, her blood pressure was checked and found to be 194/110 mm Hg. These episodes can occur with activity or at rest and she denies known precipitating events. On examination, blood pressure is 120/88 mm Hg, pulse is 82/min, temperature 98.6°F, and respirations 14/minute. Physical assessment, including cardiovascular and neurologic examination, is unremarkable. A 24-hour urine specimen as well as plasma level obtained from the patient is collected and found to have three times the expected amount of both metanephrines and free catecholamines. Question What is the most likely diagnosis? 1 Hyperthyroidism 2 Anxiety attack 3 Pheochromocytoma 4 Renal artery stenosis 5 Coronary artery disease
3 Pheochromocytoma Explanation Headache, palpitations, and diaphoresis are the classic triad of a pheochromocytoma crisis. These catecholamine-secreting tumors, most commonly found in young to mid-adult women, present with paroxysmal episodes in over 50% of patients. The episodes are of sudden onset, last minutes to hours, and they sometimes occur as a result of activity affecting the abdomen. They often occur without a clear precipitating event. Hypertension with tachycardia occurs in most patients, presenting only during attacks in 40% of those patients, while the remaining 60% have sustained hypertension with half of those patients also experiencing hypertensive crises. In the absence of sustained hypertension or a paroxysmal episode, physical exam is often unremarkable. Diagnosis is made based on a 24-hour urine specimen for vanillylmandelic acid (VMA), metanephrines, and free catecholamines. Urine collection should occur when the patient is hypertensive or initiated at the time of a paroxysmal crisis. Associated symptoms during a paroxysmal episode may include apprehension or a sense of doom suggesting an anxiety attack. Although anxiety attacks present as recurrent, unpredictable episodes of intense fear that can be associated with palpitations and sweating, they lack the malignant hypertensive response of a pheochromocytoma. A pheochromocytoma crisis may precipitate chest pain with nausea, vomiting, pallor, and arrhythmias. The catecholamine surge, rather than coronary artery disease, may induce myocardial ischemia, resulting in angina and myocardial infarction. Signs and symptoms of increased metabolic rate associated with a pheochromocytoma such as weight loss, palpitations, and sweating may also suggest hyperthyroidism. The most common cardiac manifestation in thyrotoxicosis is an arrhythmia, such as sinus tachycardia or atrial fibrillation in patients >50 years of age, and not the malignant hypertension associated with a pheochromocytoma. Renal artery stenosis presents as an abrupt onset of hypertension, malignant and/or refractory, typically associated with abdominal and/or flank bruits. When it occurs in younger women, it is most often the result of fibromuscular dysplasia of the arterial wall. When the onset occurs at >50 years of age, it is most often the result of atherosclerotic plaques.
A 21-year-old G2P1 woman presents at 10 weeks gestation with vaginal bleeding and lower abdominal cramping. She is concerned about losing her baby. Examination demonstrates bright red blood coming through a closed cervical os. Question What is the most likely diagnosis? 1 Inevitable abortion 2 Complete abortion 3 Threatened abortion 4 Incomplete abortion 5 Habitual abortion
3 Threatened abortion Explanation The clinical picture is suggestive of a threatened abortion. About 75% of abortions occur before the 16th week of gestation. In a threatened abortion, bleeding and cramping occur, but the cervix is not dilated. The pregnancy continues. In a complete abortion, the placenta and fetus are completely expelled. In an inevitable abortion, the cervix is dilated and bleeding is present. In an incomplete abortion, mild cramps are reported, but some of the products of conception—usually the placenta—remain in the uterus. The cervical os is opened. Habitual (recurrent) abortion is defined as a loss of 3 or more pre-viable pregnancies in succession.
A 62-year-old man presents with persistent hematuria. He denies any dysuria, urgency, or frequency. The first episode of hematuria was discovered 4 months ago on a routine urinalysis for a workplace physical. A repeat urinalysis 6 weeks later again shows hematuria, confirmed by microscopic evaluation. The remainder of his urinalysis is within normal limits. His past medical history is remarkable for COPD and obesity. He has smoked 1.5 ppd x 45 years. He uses inhaled medications for his COPD and has NKDA. His physical exam is significant for a temperature of 100.5°F. Abdominal exam elicits mild tenderness in the left upper quadrant, but no masses are palpable (although his exam is limited by his obesity). Chest and abdominal CTs with contrast show a normal chest and a left renal mass enhanced by radiocontrast, suggestive of renal cell carcinoma (RCC). No metastatic disease was noted on imaging. A renal biopsy is pending. Question Assuming the biopsy supports the diagnosis of renal cell carcinoma, what referral would be most appropriate for this patient? 1 Medical oncologist for chemotherapy 2 Radiation oncologist for radiation therapy 3 Urologist for radical nephrectomy 4 Geneticist for associated cancer screening 5 Hospice for pain management and palliative care
3 Urologist for radical nephrectomy Explanation The most appropriate intervention for this patient is referral to a urologist for radical nephrectomy, the standard treatment for localized RCC. The nephrectomy serves to diagnose, stage, and treat the cancer. A referral to a medical oncologist for traditional cytotoxic chemotherapy is not recommended, as RCC is refractory to traditional chemotherapies. Immunotherapy (interleukin or interferon) and antiangiogenic agents are considered a reasonable addition to nephrectomy in patients with metastatic RCC, but there is no current evidence suggesting this patient has metastatic disease. RCC is considered a radiation-resistant tumor, so external beam radiation is rarely used as a primary treatment. Radiation may have a role in palliative treatment in special circumstances. Referral to a geneticist for associated cancer screening is a reasonable step to take in the process of the oncologic workup, as there are known familial causes of RCC and genetic mutations associated with RCC, although RCC is far more often a sporadic occurrence. A referral to hospice for pain management and palliative care only suggests there are no reasonable treatments for this patient and that he is expected to die within 6 months; 5-year survival rates are around 66% for stage I RCC.
A 34-year-old woman presents with a 6-month history of progressive fatigue. She has had to stop running due to lack of energy, and she notes that she is napping frequently. She also has a poor appetite, dry skin, and darkening of the skin over her elbows and knees. Physical examination is significant for orthostatic hypotension and hyperpigmentation of the elbows and knees. Basic metabolic panel reveals the following: Component Value Range Sodium 125 135-146 mmol/L Potassium 5.3 3.5-5 mmol/L Chloride 78 98-107 mmol/L Bicarbonate 23 23-32 mmol/L Glucose 95 65-100 mg/dL BUN 17 8-25 mg/dL Creatinine 0.7 0.8-1.30 mg/dL Question What testing is recommended to confirm her diagnosis? 1 Dexamethasone suppression test 2 Plasma fractionated 3 Metanephrine 4 Cosyntropin stimulation test 5 Radioactive iodine uptake test
4 Cosyntropin stimulation test Explanation The patient's history, presence of hyperpigmentation, and presence of hyperkalemia are suggestive of Addison's disease, or primary adrenal insufficiency. The cosyntropin stimulation test (also called the ACTH test) is used to assess the function of adrenal gland response. A small amount of cosyntropin (synthetic ACTH) is injected, and the serum cortisol is obtained 45 minutes following administration. Normally, the cortisol level should respond to cosyntropin injection. Patients with Addison's disease lack the normal response to ACTH, so the cortisol level will fail to rise. Dexamethasone suppression test is used in the diagnosis of Cushing syndrome, which is caused by hypercortisolism. Plasma fractionated metanephrine are used in the diagnosis of pheochromocytoma. Serum CA 19-9 is a tumor marker for pancreatic cancer. The radioactive iodine uptake test is used in the work-up of hyperthyroidism.
A 60-year-old man presents with difficulty climbing stairs, dyspnea, and fatigue. He has gained 30 lb over the past year. On examination, he is found to have edema, pigmentation of the skin, palmar creases, and proximal muscle weakness. Chest X-ray shows an irregular mass in the right upper lobe. Lab values show an increase in evening cortisol levels and an increase in ACTH. Question What should be the next best step in diagnosis? 1 MRI of the brain 2 Adrenal venous sampling 3 Repeat serum ACTH levels 4 Dexamethasone suppression test 5 ACTH stimulation test
4 Dexamethasone suppression test Explanation Lab values are indicative of raised cortisol and an increased ACTH level. The next step in diagnosis should be to differentiate between a pituitary and peripheral cause of high ACTH. A high-dose dexamethasone suppression test is used to differentiate between Cushing syndrome caused by pituitary adenoma and ectopic ACTH production by a lung mass. ACTH of pituitary origin is suppressed by high doses of dexamethasone by negative feedback; ACTH of peripheral origin is not. Adrenal venous sampling is of great utility in patients with primary hyperaldosteronism. MRI of the brain is appropriate after an endocrine diagnosis is made. If done prior, an unacceptably high false positive rate ensues. ACTH levels are already available; no repeat is necessary for the next step in diagnosis. ACTH stimulation test is used in adrenal insufficiency.
A 55-year-old woman presents with a slight cough she has had for about a week. She is a nonsmoker, and she does not remember having a fever or feeling sick. The patient currently works as a third-grade teacher and has done this for 30 years. She has no past medical history of significant pulmonary diagnoses. Auscultation of the chest reveals clear lung fields. A chest X-ray shows a subpleural "coin lesion" in the right upper lobe. Question What is the most likely diagnosis? 1 Small cell anaplastic carcinoma 2 Bronchiectasis 3 Silicosis 4 Granuloma 5 Exogenous lipid pneumonia
4 Granuloma Explanation The most likely diagnosis is granuloma. The differential diagnosis of a solitary coin lesion includes: Lung carcinoma (most commonly an adenocarcinoma) Granuloma Hamartoma A small cell carcinoma (or oat cell carcinoma) tends to spread very quickly and does not remain localized. Moreover, it is most common with a history of smoking. Silicosis results from years of inhaling dust containing silica, and it produces diffusely scattered nodules in the lungs. Bronchiectasis results from inflammation, destruction, and dilatation of the bronchi; it does not produce a round discreet lesion. Exogenous lipid pneumonia is due to aspiration of a substance that is oily or contains lipids. It is not typically localized.
You transfuse 3 units of whole blood to a gunshot victim. Unknown to you, there was a shortage of blood, and the units you gave to your patient were from the bottom of the freezer and about to expire. Within 2 days, the transfused cells are breaking down, and your patient's ECG is beginning to show high T waves and a widened QRS complex. Question According to the condition being described above, which of the following is evident on EKG? 1 Hyponatremia 2 Hypernatremia 3 Hypokalemia 4 Hyperkalemia 5 Hypocalcemia
4 Hyperkalemia Explanation Due to the mechanism of breakdown of the transfused cells, the patient's EKG is showing evidence of hyperkalemia. This is evident by the EKG displaying peaked T waves and widened QRS complex. Hypokalemia is incorrect. EKG findings of this state would be: decreased amplitude and broadening of T waves, prominent U waves, premature ventricular contractions, or even depressed ST segments. Significant hyponatremia may be evident by ST-segment elevation, perhaps mimicking acute myocardial infarction. Hypernatremia is evident by sinus tachycardia, shortened PR intervals, and diffuse ST depressions. Hypocalcemia may result in prolongation of the QT interval due to the lengthened ST segment.
A 42-year-old woman delivers her third child while you are on call at a local hospital on the Atlantic coast. She thanks you for your help and mentions that she is grateful that her baby was on time. She states that the family is planning on taking their newest with them to Colorado in 2 weeks. You advise her not to attempt the trip, as the baby will have difficulty adjusting to the high altitudes. Question What factor is a major component in the closure of the ductus arteriosus? 1 Increase in pulmonary arterial pressure associated with increase in blood flow to the lungs 2 Reduction of aortic arterial pressure due to reduction in systemic resistance 3 Decrease in carbon dioxide levels in the blood 4 Increase in oxygen tension within the blood 5 Presence of prostaglandin E2 in the wall of the ductus arteriosus
4 Increase in oxygen tension within the blood Explanation Increase in oxygen tension is thought to be the primary initiator of the closure of the ductus arteriosus. The pO2 increases from 15 to 20 mm Hg to around 100 mm Hg in a few hours. Increase in pulmonary arterial pressure associated with increased blood flow to the lungs is not the major component in the closure of the ductus arteriosus. Pulmonary pressure and pulmonary blood pressure decreases, resulting in a decreased flow from the pulmonary circuit to the systemic circuit. Reduction of aortic arterial pressure due to reduction in systemic resistance is not a major component in closure of the ductus arteriosus. After birth, systemic blood pressure rises as a result of increased effective blood volume and decreased pulmonary resistance. The ductus no longer serves as a shunt from pulmonary to systemic circulation. As gas exchange increases, the amount of carbon dioxide in the neonate will decrease. However, this does not initiate vasoconstriction and closure of the ductus arteriosus. Prostaglandins are thought to be responsible for maintaining patency of the ductus arteriosus.
A 29-year-old woman has Wilson's disease, for which she is taking D-penicillamine. Today, her laboratory findings (including liver function tests) are within normal limits; there are no neurological signs. Her work requires frequent (3 - 5 days per week) travel, including international overnight travel all over the world. She wants contraception that is both effective and convenient in terms of application. Question What is your advice? 1 Combined oral contraceptive pill 2 Progestin-only pill 3 Nonhormonal copper intrauterine device 4 Levonorgestrel-releasing Intrauterine system 5 Contraceptive patch
4 Levonorgestrel-releasing Intrauterine system Explanation Only spermicide, barrier contraceptives, and progesterone-only preparations can be considered in Wilson's disease. Levonorgestrel is a 2nd-generation progestin (synthetic progestogen). Alevonorgestrel-releasing intrauterine system causes thickening of cervical mucus, thereby reducing sperm motility and penetration; over time it decreases proliferation of the endometrium. The local inflammatory response could be toxic to sperm, thus providing an additional contraceptive effect. Combination oral contraceptive pills contain both estrogen and progestins. Estrogen-containing contraceptives are not recommended since estrogen may interfere with copper excretion. The progestogen-only pill is efficient; however, it must be taken at or around the same time every day (within 3 - 12 hours). Therefore, the pill may be inconvenient for a woman with a job that requires international travel all over the world. Nonhormonal copper intrauterine devices should be avoided in a disease that has the main characteristic of copper accumulation in the body. Copper IUDs primarily work by disrupting sperm mobility and damaging sperm. Contraceptive patches also contain estrogen; therefore, they should be avoided in Wilson's disease.
A 73-year-old man presents with the inability to actively raise his left non-dominant arm to retrieve plates from the kitchen cabinet. This began a month ago after his shoulder pain improved. He had a history of pain in that shoulder for over 6 months that kept him from sleeping on the left side and the pain would wake him often. There was no specific injury he can recall although he felt a pop a month ago while taking out the trash. What is the diagnostic study of choice if surgery is indicated? 1 CT without contrast 2 Electromyelogram (EMG) 3 Ultrasound 4 MRI 5 Plain films
4 MRI Explanation This elderly man probably has a torn rotator cuff. If surgical treatment is being considered, MRI is the imaging study of choice because it can provide additional information on the status of the muscle and the size of the rotator cuff tear. Electromyelogram is used to assess the nerves. Ultrasound doesn't give as clear a picture of the tear due to the bony interference. Plain films can assess only the boney picture.
A 62-year-old woman presents with extreme fatigue and shortness of breath. The symptoms began about 24 hours ago and have progressively worsened within the last 4 hours. Vital signs on arrival are as follows: HR 90 beats per minute; BP 165/72 mm Hg; RR 16/min; SpO2 98% on 4 L/min supplemental oxygen by nasal cannula. 12-lead ECG demonstrates ST-segment elevation of 2 mm in leads V4-V6. Question In addition to an aspirin tablet, what medication would be most appropriate in the emergency management of this patient? 1 Dobutamine 2 Dopamine 3 Morphine 4 Nitroglycerin 5 Vasopressin
4 Nitroglycerin Explanation The patient's presentation is consistent with acute myocardial infarction. Emergency department management of patients with acute coronary syndromes, which include acute myocardial infarction and unstable angina, should consist of supplemental oxygen to maintain SpO2 >90%, oral aspirin 160-325 mg, and sublingual nitroglycerin unless contraindicated (for example, in the context of hypotension). The purpose of nitroglycerin is to reduce myocardial oxygen demand; this alone usually relieves chest discomfort if present. For patients with pain that is not alleviated by nitroglycerin, administration of intravenous morphine may be considered. Vasopressin is a vasoconstrictor formerly considered as an alternative medication in cardiac arrest refractory to epinephrine therapy. As of 2015, however, vasopressin was removed from the treatment algorithms for ventricular fibrillation/tachycardia and asystole/pulseless electrical activity, owing to lack of evidence that it benefited patients. Dobutamine and dopamine are inotropes used to improve cardiac output and support end-organ perfusion in the setting of shock. None of these medications are part of the treatment algorithm for acute coronary syndrome.
A patient recovering from hip surgery begins to ambulate for the first time about 2 hours postoperatively. Suddenly, they experience shortness of breath. The patient becomes tachypneic and tachycardic and experiences pain on inspiration. Prior to getting out of bed, their postoperative course was unremarkable. There is no swelling; there is no palpable thrill at the incision site. Question What is your initial diagnosis? 1 Postoperative pneumonia 2 Atelectasis 3 Arteriovenous fistula at surgical site 4 Pulmonary embolism 5 Abdominal aortic aneurysm rupture
4 Pulmonary embolism Explanation The clinical picture is suggestive of a pulmonary embolism. Pulmonary embolisms are a serious and potentially fatal complication of thrombi formation, usually in the lower extremities. Risk factors include venous stasis, hypercoagulability, and injury to the vessel wall; they are known as Virchow's triad. Venous stasis increases with immobility—bed rest, in this case—especially postoperatively. Clinical findings include dyspnea, chest pain, syncope, hemoptysis, tachypnea, and pain on inspiration. Postoperative pneumonia usually occurs 48 hours postoperatively, and the patient will present with fever on day 3 or 4 postoperatively. Atelectasis is a common postoperative complication (collapse of lung tissue), especially after chest or abdominal surgery. Arteriovenous fistula is an abnormal connection between an artery and a vein; a palpable thrill can be felt at the incision site. An abdominal aortic aneurysm rupture presents with severe pain, a palpable abdominal mass, and hypotension.
A 44-year-old woman undergoes surgery for a follicular neoplasm of the thyroid gland. The tumor is successfully resected, and she is transferred to the post-operative ward. Her condition is noted to be stable. On postoperative day 2, she notices a tingling sensation around her lips that runs down her arms. She experiences a cramping sensation in the muscles of her hands and legs, with her fingers going into spasm. An intern on the floor notices the patient's distress and decides to investigate. Question What is the best initial diagnostic test? 1 Serum free T3 and T4 2 Serum TSH 3 Arterial blood gas analysis 4 Serum calcium levels 5 CT scan of the anterior neck
4 Serum calcium levels Explanation The patient has symptoms of hypocalcemia, a possible complication of thyroid surgery. The parathyroid glands are closely related to the thyroid gland and may be inadvertently removed during surgery. If damaged, the serum calcium levels usually drop within 48 hours of surgery. Symptoms of hypocalcemia include muscle pain, fatigue, and muscle cramps if severe. Chvostek's sign, which is the contraction of ipsilateral facial muscles with a light tap over the facial nerve, and spasm of the carpal muscles (Trousseau's sign) may be seen. The best screening test for hypocalcemia is the measurement of ionized calcium in the serum. Calcium exists in equilibrium between the fraction bound to albumin and the free or ionized fraction. The biological effects of calcium are dependent on the ionized fraction. Serum calcium level varies with level of serum albumin (calcium binding protein). Correction for calcium level when albumin is abnormal is as follows: Corrected calcium (mg/dL) = measured total Ca (mg/dL) + 0.8 (4.0 - serum albumin [g/dL]), where 4.0 represents the average albumin level. The risk factors for hypocalcemia after thyroid surgery include large goiters, hyperthyroidism, low preoperative vitamin D levels, and planned extensive neck dissection. Hypocalcemia is prevented to some degree by the administration of large amounts of calcium, usually per oral, and 1,25 dihydroxy vitamin D. In the event of severe hypocalcemia, calcium gluconate may be used via IV. It is important to check magnesium levels and correct levels prior to discharge, as hypomagnesemia can compound hypocalcemia and make it refractory to correction. Serum TSH, T3, and T4 are incorrect, as thyroid function has no bearing on the patient's symptoms. They are expected to change after thyroid surgery. Typically, TSH levels will rise, and T3 and T4 levels will fall over the ensuing 3-6 weeks. An ABG is incorrect. It may demonstrate low Ca levels, but a direct measure of serum calcium in patients that demonstrates symptoms of hypocalcemia is more accurate and effective. A CT scan of the neck is incorrect, as it provides no additional information about the person's calcium metabolism.
A 48-year-old Caucasian man presents with acute onset of blurring of vision and severe pain in the left eye that began 30 minutes ago. He notes seeing halos with his left eye. He is also experiencing nausea and vomiting. These symptoms started at the same time as the pain. The patient reports that he was relaxing on his porch when the pain started. His temperature is 36.9°C, pulse 90/min, BP 130/90 mm Hg, and respirations 20/min. Physical examination reveals a shallow anterior chamber, a hazy cornea, a fixed, moderately dilated pupil, and ciliary injection. Question What would be the next step in the management of this patient? 1 Lumbar puncture 2 X-ray to rule out a foreign body 3 Topical atropine to the eye to facilitate ophthalmoscopy 4 Tonometry 5 Discharge with topical antibiotic drops for the eye
4 Tonometry Explanation The history and physical examination of this patient are suggestive of acute angle-closure glaucoma, which can be easily confirmed by measuring the intraocular pressure using a tonometer (e.g., the Schiötz tonometer). Acute angle-closure glaucoma develops in individuals with pre-existing anatomic narrowing of the anterior chamber, which is seen mainly in far-sighted people. The condition usually develops in the twilight hours, which is when the pupil is dilated in response to the low level of illumination. It may also occur with pupillary dilation for ophthalmoscopy, so topical atropine would be contraindicated. Acute angle-closure glaucoma is an ophthalmologic emergency. Treatment involves immediate lowering of the intraocular pressure via systemic acetazolamide to decrease the production of aqueous humor, which should be supplemented with topical hyperosmotic agents and topical beta blockers. Topical pilocarpine is then used to cause miosis. Once the intraocular pressure is under control, a peripheral iridectomy can be done to prevent against future recurrences.
A 20-year-old female college student presents due to a 7-day history of daily heartburn. She has never had this as bad as she does currently. The patient denies any other significant past medical history and is currently taking only a multivitamin daily. She admits to recently having increased episodes of headaches that she believes are due to stress. For this reason, she has been taking ibuprofen 600 mg every 8 hours. She states that she has been taking this consistently every 8 hours for the last 10 days. She hopes that after finals are over her headaches will subside. She also states she was given a 10-day course of amoxicillin 2 weeks ago for a middle ear infection, which resolved without any further intervention. She denies any difficulty swallowing, weight loss, night sweats, chest pain, black tarry stools, use of tobacco/alcohol, or coughing up blood. Physical examination is unremarkable for any abnormalities. Question What medication would you suggest for the patient to begin for her headaches at this time? 1 Antibiotics 2 Anticholinergic 3 Proton pump inhibitor 4 H1-receptor antagonist 5 Acetaminophen
5 Acetaminophen Explanation Patients classified as having mild or intermittent symptoms of esophagitis and/or gastroesophageal reflux disease (GERD) typically are seen as not adversely affecting the patient's quality of life. Initial action is taken in terms of modification of behaviors, such as eating smaller meals and eliminating acidic foods or foods known to precipitate reflux (fatty foods, alcohol, chocolate, or peppermint). The next step in terms of treatment would be to discontinue any medications that may be increasing the symptoms of GERD. Medications that potentially irritate the esophagus and have a high incidence of causing pill-induced esophagitis include tetracycline, bisphosphonates, iron supplements, Non-Steroidal Anti-inflammatories (NSAIDs), and potassium supplements. Medications that can increase acid reflux and worsen the condition of GERD include anticholinergics, calcium channel blockers, narcotics, progesterone, quinidine, benzodiazepines, or even theophylline. A proton pump inhibitor would be initiated in a patient who has moderate to severe esophagitis or gastroesophageal reflux symptoms; this patient does not fit this picture. Since this patient is currently choosing ibuprofen chronically for a recent onset of headaches, the appropriate instruction would be to discontinue the ibuprofen regimen and begin acetaminophen regimen in its place.Antibiotics would be an incorrect answer. H1 receptor antagonists, or H1-antihistamines, are an inappropriate choice because these are used to treat the symptoms of allergies (seasonal and perennial). The mechanism of action of anticholinergics does not make it an indication in this type of medical complaint.
A 40-year-old obese Caucasian man says that his father recently passed away after having a heart attack. He is worried and wants to know whether he is at risk for cardiovascular disease. He is hypertensive; however, he does not take any medications. His BP in the office today is 140/96 mm Hg, and his BMI is 31. You explain to him that the risk factors for cardiovascular disease may be non-modifiable or modifiable. Question What is the modifiable risk factor for CAD in this man? 1 Age 2 Ethnicity 3 Sex 4 Family history 5 Hypertension
5 Hypertension Explanation Men are at higher risk of CAD than women. Women are at lower risk due to the cardioprotective property of estrogen; therefore, the risk increases after menopause. Increasing age is a risk factor for CAD; age above 45 years for men and 55 years for women is considered a risk factor. The non-modifiable risk factors include increasing age, male sex, family history of CAD, and ethnicity. African Americans, Native Americans, Mexican Americans, and some Asian groups are at greater risk than others, partly due to the higher incidence of hypertension, diabetes, and obesity among these groups. The modifiable risk factors include diabetes, hypertension, smoking, and blood lipid levels. A sedentary lifestyle and fat- or salt-rich diet are also considered risk factors.
A 45-year-old Caucasian woman presents with a 2-month history of a lump in her right breast. She has had no children. Her menstrual periods began at age 15 and are still regular. She is a successful businessperson who does not smoke cigarettes and drinks 6 cups of coffee per day. She gets 30 minutes of physical activity at least 5 times weekly. On examination, she has a nontender lump in the upper outer quadrant of her right breast. Several matted right axillary lymph nodes are also palpable. Question Given her history, what is her greatest risk factor for developing breast cancer? 1 Regular physical activity 2 Menarche at 15 years old 3 Menopause at age 45 years old 4 Excessive caffeine consumption 5 Nulliparity
5 Nulliparity Explanation The correct response is nulliparity. The clinical features are suggestive of breast cancer. Although it occurs mainly in women, a small percentage develops in men. Breast cancer has no single cause, but multiple risk factors have been identified. These include: Family history of premenopausal breast cancer in a first-degree relative (e.g., mother, daughter, sister) Having the first full-term pregnancy after age 30 Prior personal history of breast cancer Chest exposure to high dose of ionizing radiation Nulliparity Early menarche at or before age 12 Late menopause after age 50 Taking oral contraceptives (birth control pills) Lack of physical activity Current or recent use of oral contraceptive pills (for women who stopped using oral contraceptive pills 10+ years ago, there is no apparent increase in risk).